submerged in inert oil. An oxygen value of 10 mm Hg is recorded. The
distal duodenum at the same location is then treated with the
vasodilator, adenosine. Which value of oxygen is most likely in the tip
of the villus within 2 minutes after treatment with adenosine (in mm
Hg)?
A) 0
B) 5
C) 7
D) 10
E) 12
57. One of the following hormones can stimulate growth of the intestinal
mucosa, and two other hormones can stimulate pancreatic growth.
Which three hormones are these?
Gastrin Secretin Cholecystokinin GIP Motilin
A) No Yes Yes Yes No
B) Yes No Yes No Yes
C) Yes No Yes Yes No
D) Yes No Yes Yes No
E) Yes Yes Yes No No
58. A 65-year-old man eats a healthy meal consisting of 30%
carbohydrates, 20% fats, and 50% proteins. Approximately 40 minutes
later, the ileocecal sphincter relaxes, and chyme moves into the cecum.
Gastric distention leads to relaxation of the ileocecal sphincter by way
of which reflex?
A) Enterogastric
B) Gastroileal
C) Gastrocolic
D) Intestino-intestinal
E) Rectosphincteric
59. The gastric mucosal barrier has a physiological and an anatomical basis
to prevent hydrogen ion accumulation in the mucosa. Some factors are
known to strengthen the integrity of the gastric mucosal barrier,
whereas other factors can weaken the barrier. Which factors strengthen
or weaken the barrier?
Bile Salts Mucous Aspirin NSAIDs Gastrin Ethanol
A) Strengthen Strengthen Weaken Weaken Strengthen Strengthen
B) Strengthen Strengthen Weaken Weaken Weaken Strengthen
C) Weaken Strengthen Strengthen Weaken Strengthen Weaken
D) Weaken Strengthen Weaken Weaken Strengthen Weaken
E) Weaken Weaken Weaken Streng-then Strengthen Weaken
60. The assimilation of fats includes (1) micelle formation, (2) secretion of
chylomicrons, (3) emulsification of fat, and (4) absorption of fat by
enterocytes. Which sequence best describes the correct temporal order
of these events?
A) 4, 3, 2, 1
B) 3, 1, 4, 2
C) 3, 4, 1, 2
D) 2, 1, 4, 3
E) 4, 2, 1, 3
F) 2, 4, 1, 3
G) 1, 2, 3, 4
H) 1, 3, 2, 4
61. A 62-year-old man with dyspepsia and a history of chronic gastric
ulcer has abdominal pain. Endoscopy shows a large ulcer in the
proximal gastric body. Biopsies are positive for H. pylori. Which
substances are used clinically for treatment of gastric ulcers of various
etiologies?
Antibiotics NSAIDs H2 Blockers Proton Pump Inhibitors
A) No No Yes Yes
B) Yes No No Yes
C) Yes No Yes Yes
D) Yes Yes Yes Yes
E) No Yes Yes Yes
62. Cystic fibrosis (CF) is an inherited disorder of the exocrine glands
affecting children and young people. Mucus in the exocrine glands
becomes thick and sticky and eventually blocks the ducts of these
glands (especially in the pancreas, lungs, and liver), forming cysts. A
primary disruption in the transfer of which ion across cell membranes
occurs in CF, leading to decreased secretion of fluid?
A) Calcium
B) Chloride
C) Phosphate
D) Potassium
E) Sodium
63. A 43-year-old man presents with abdominal pain and hematemesis. An
abdominal examination was relatively benign, and abdominal
radiographs were suggestive of a perforated viscus. Endoscopy
revealed a chronically perforated gastric ulcer, through which the liver
was visible. Which mechanism is a forerunner to gastric ulcer
formation?
A) Back-leak of hydrogen ions
B) Mucus secretion
C) Proton pump inhibition
D) Tight junctions between cells
E) Vagotomy
64. A 12-year-old girl consumes a glass of milk and two cookies. Her lower
esophageal sphincter (LES) and fundus relax while the food is still in
the esophagus. Which substance is most likely to cause relaxation of the
LES and fundus in this girl?
A) Gastrin
B). Histamine
C) Motilin
D) Nitric oxide
E) Norepinephrine
65. Mass movements can be stimulated after a meal by distention of the
stomach (gastrocolic reflex) and distention of the duodenum
(duodenocolic reflex). Mass movements often lead to which of the
following?
A) Bowel movements
B) Gastric movements
C) Haustrations
D) Esophageal contractions
E) Pharyngeal peristalsis
66. A 45-year-old woman with type 1 diabetes has an early feeling of
fullness when eating. She is often nauseous after a meal and vomits
about once each week after eating. Glucose-induced damage to which
structure is most likely to explain her gastrointestinal problem?
A) Celiac ganglia
B) Enteric nervous system
C) Esophagus
D) Stomach
E) Vagus nerve
67. Which stimulus–mediator pair normally inhibits gastrin release?
Stimulus Mediator
A) Acid CCK
B) Acid GIP
C) Acid Somatostatin
D) Fay acid Motilin
E) Fay acid Somatostatin
68. A 55-year-old man consumes a meal consisting of 20% fat, 50%
carbohydrate, and 30% protein. The following gastrointestinal
hormones are released at various times during the next 6 hours: gastrin,
secretin, motilin, glucose-dependent insulinotropic peptide, and
cholecystokinin. Which structure is most likely to release all five
hormones in this man?
A) Antrum
B) Colon
C) Duodenum
D) Esophagus
E) Ileum
69. A 79-year-old man has a cerebrovascular accident (stroke) in the
medulla and pons that eliminates all vagal output to the gastrointestinal
tract. Which function is most likely to be eliminated in this man?
A) Gastric acid secretion
B) Gastrin release
C) Pancreatic bicarbonate secretion
D) Primary esophageal peristalsis
E) Secondary esophageal peristalsis
70. A 74-year-old man with hematemesis and melena is diagnosed with a
duodenal ulcer. Which of the following is most likely in this patient?
Parietal Cell Density Acid Secretion Plasma Gastrin
A) Decreased Decreased Decreased
B) Decreased Increased Decreased
C)
Increased Decreased Increased
D)
Increased Increased Decreased
E)
Increased Increased Increased
71. A 61-year-old man with upper abdominal pain and blood in the stool
takes NSAIDs for the pain and washes it down with vodka.
Pentagastrin administration produced lower than predicted levels of
gastric acid secretion. Secretion of which substance is most likely to be
diminished in this patient with gastritis?
A) Intrinsic factor
B) Ptyalin
C) Rennin
D) Saliva
E) Trypsin
72. Gastric acid is secreted when a meal is consumed. Which factors have a
direct action on the parietal cell to stimulate acid secretion?
Gastrin Somatostatin Acetylcholine Histamine
A) No No Yes Yes
B) Yes No No Yes
C) Yes No Yes Yes
D) Yes Yes Yes Yes
E) Yes Yes No Yes
73. A 37-year-old woman adds high-fiber wheat and bran foods to her diet
to reduce her serum cholesterol levels. She had avoided eating foods
containing wheat or rye since she was a child because her mother said
they would make her sick. The woman loses 25 lb on her new diet but
has frequent stomach cramps, gas, and diarrhea. She has also become
weaker, finding it difficult to complete her morning walks. What is most
likely to be increased in this woman?
A) Blood hemoglobin concentration
B) Carbohydrate absorption
C) Fecal fat
D) Protein absorption
E) Serum calcium
74. A newborn boy does not pass meconium within 48 hours of delivery.
His abdomen is distended, and he begins vomiting. A suction biopsy of
a distally narrowed segment of the colon shows a lack of ganglionic
nerve cells. This newborn is at risk for developing which condition?
A) Achalasia
B) Enterocolitis
C) Halitosis
D) Pancreatitis
E) Peptic ulcer
75. A 41-year-old obese woman with a history of gallstones is admied to
the emergency department because of excruciating pain in the upper
right quadrant. The woman is jaundiced, and a radiograph suggests
obstruction of the common bile duct. Which values of direct and
indirect bilirubin are most likely to be present in the plasma of this
woman (in milligrams per deciliter)?
Direct Indirect
A) 1.0 1.3
B) 2.3 2.4
C) 5.0 1.7
D) 1.8 6.4
E) 6.8 7.5
76. Which mechanism for transport of substances across the luminal cell
membrane of a duodenal enterocyte is present in newborns and infants
but not in adults?
A) Endocytosis
B) Facilitated diffusion
C) Passive diffusion
D) Primary active transport
E) Secondary active transport
77. Cystic fibrosis is the most common cause of pancreatitis in children.
Which option best explains the mechanism of cystic fibrosis-induced
pancreatitis?
A) Activation of enterokinase
B) Activation of trypsin inhibitor
C) Autodigestion of pancreas
D) Excessive secretion of CCK
E) Gallstone obstruction
Answers
1. B) Gastrin is secreted mainly by the G-cells of the antrum of the stomach.
The primary actions of gastrin are (1) stimulation of gastric acid secretion
and (2) stimulation of mucosal growth throughout the gastrointestinal
tract.
TMP14 p. 792
2. D) Motilin is secreted by the duodenum and jejunum during fasting, and
the only known function of this hormone is to increase gastrointestinal
motility. Motilin is released cyclically and stimulates waves of
gastrointestinal motility called interdigestive myoelectric complexes (or
migrating motility complexes) that move through the stomach and small
intestine every 90 minutes in a person who has fasted. The purpose of
these interdigestive myoelectric complexes is to remove food residue from
the intestine, which lowers bacterial growth. Hence, interdigestive
myoelectric complexes are also called housekeeping contractions.
TMP14 pp. 792–793
3. B) Gastrin is secreted in response to vagal stimulation as well as stimuli
associated with ingestion of a meal, such as distention of the stomach and
the breakdown products of proteins. However, fat does not cause the
release of gastrin as it does with other gastrointestinal hormones.
TMP14 p. 792
4. D) Under normal conditions, the blood flow of the gastrointestinal tract is
directly related to the level of local activity. For instance, after a meal, the
motor activity, secretory activity, and absorptive activity all increase;
likewise, the blood flow increases greatly. This increase in blood flow is
caused by vasodilation of intestinal blood vessels. The increase in cardiac
output associated with consuming a large meal is causing ischemic chest
pain in this man.
TMP14 pp. 794–795
5. D) Sodium ions are actively reabsorbed from the salivary ducts, and
potassium ions are actively secreted in exchange for the sodium. When
salivary flow is elevated, each unit portion of saliva spends less time in the
salivary ducts. Hence, there is less time for absorption of sodium
(increased sodium concentration) and less time for potassium secretion
(decreased potassium concentration).
TMP14 p. 808
6. D) This student has temporary lactose intolerance resulting from the
temporary loss of the enzyme lactase. Lactase is a brush border enzyme; its
production can be temporarily depressed following an intestinal viral
infection (viral gastroenteritis).
TMP14 p. 824
7. F) The presence of acid, fat, protein breakdown products, hyperosmotic or
hypo-osmotic fluids, or any irritating factor in the upper small intestine
causes release of several intestinal hormones. One of these hormones is
secretin, which is especially important for stimulating pancreatic
bicarbonate secretion. However, secretin opposes gastric acid secretion.
Three other hormones—glucose-dependent insulinotropic peptide
(formerly called gastric inhibitory peptide), vasoactive intestinal
polypeptide, and somatostatin—also have slight to moderate effects in
inhibiting gastric acid secretion.
TMP14 pp. 792, 814
8. E) The same cells that secrete proteolytic enzymes into the acini of the
pancreas simultaneously secrete another substance called trypsin inhibitor;
this substance prevents activation of trypsin both inside the secretory cells
and in the acini and ducts of the pancreas. When the pancreas becomes
severely damaged or when a duct becomes blocked, large quantities of
pancreatic juice can become pooled in the pancreas. Under these
conditions, the effect of trypsin inhibitor is often overwhelmed, in which
case the pancreatic secretions rapidly become activated and can literally
digest large portions of the pancreas within a few hours, giving rise to the
condition called acute pancreatitis. The obese woman described in this
question has gallstone obstruction at the sphincter of Oddi, which causes
pancreatic juice to pool within the pancreas, leading to autodigestion of the
pancreas.
TMP14 pp. 815, 835
9. C) Fructose transport does not occur by the sodium co-transport
mechanism used for glucose and galactose absorption. Instead, fructose is
transported by facilitated diffusion all the way through the intestinal
epithelium and is not coupled with sodium transport.
TMP14 p. 831
10. A) When liver parenchymal cells are destroyed, they are replaced with
fibrous tissue that eventually contracts around the blood vessels, raising
pressure in the entire portal vascular system of the gastrointestinal tract;
this increases capillary hydrostatic pressure. This high capillary pressure
causes fluid to transude into the abdominal cavity, creating ascites. The
formation of ascitic fluid is further increased by lower than normal levels
of albumin (decreased plasma colloid osmotic pressure) because the
production of albumin is lower than normal in the diseased liver.
TMP14 pp. 317–318, 872
11. B) Assimilation of proteins means digestion and absorption of proteins.
Protein digestion begins in the stomach with the actions of pepsin (item 2).
Next, the presence of fat and protein in the small intestine stimulates the
release of cholecystokinin (item 3). Cholecystokinin then causes the release
of proteolytic enzymes from the pancreas (item 4). The protein digestion
products are then absorbed by enterocytes of the gut wall (item 1).
TMP14 pp. 824–825
12. E) Once the vomiting center has been sufficiently stimulated and the
vomiting act has been initiated, the first effects are (1) a deep breath, (2)
raising of the hyoid bone and larynx to pull the upper esophageal
sphincter open, (3) closing of the glois to prevent vomitus flow into the
lungs, and (4) lifting of the soft palate to close the posterior nares. Next
comes a strong downward contraction of the diaphragm along with
simultaneous contraction of all the abdominal wall muscles, which
squeezes the stomach between the diaphragm and the abdominal muscles,
building the intragastric pressure to a high level. Finally, the lower
esophageal sphincter relaxes completely, allowing expulsion of the gastric
contents upward through the esophagus and into the mouth.
TMP14 pp. 837–838
13. E) The electrochemical gradient for sodium results from active transport
of sodium ions through the basolateral membranes of the enterocytes and
into the adjacent interstitial spaces. This electrochemical gradient for
sodium powers the secondary active transport of many different nutrients
into the epithelial cells of the gut wall. These nutrients include glucose,
galactose, and several different amino acids.
TMP14 pp. 828–829
14. A) Arterial flow into the villus and the venous flow out of the villus are in
directions opposite to each other, and the vessels lie in close apposition to
each other. Because of this vascular arrangement, much of the blood
oxygen diffuses out of the arterioles directly into the adjacent venules
without ever being carried in the blood to the tips of the villi. As much as
80% of the oxygen may take this short-circuit route and is therefore not
available for local metabolic functions of the villi.
TMP14 pp. 795–796
15. D) When feces enter the rectum, distention of the rectal wall initiates
afferent signals that spread through the myenteric plexus to initiate
peristaltic waves in the descending colon, sigmoid, and rectum, forcing
feces toward the anus. As the peristaltic wave approaches the anus, the
internal anal sphincter is relaxed by inhibitory signals from the myenteric
plexus; if the external anal sphincter is also consciously, voluntarily relaxed
at the same time, defecation occurs.
TMP14 pp. 805–806
16. G) When food enters the stomach, the pH of the gastric contents increases
because the food buffers the acid (item 2). This increase in pH lowers
somatostatin secretion from delta cells in the stomach wall, which leads to
an increase in the rate of acid secretion (item 3). The pH of the gastric
contents remains high until the buffering capacity of the food is saturated.
Next, the pH of the gastric contents decreases (item 1), which stimulates
the secretion of somatostatin. Somatostatin decreases the rate of acid
secretion (item 4) by a direct action on parietal cells as well as by decreased
gastrin release from G-cells.
TMP14 p. 814
17. B) Gastrin is the only gastrointestinal hormone listed that is produced and
stored in the antrum of the stomach. All five gastrointestinal hormones are
produced and stored in the duodenum and jejunum.
TMP14 p. 792
18. E) During the swallowing process, the soft palate is pulled upward to
close the posterior nares to prevent reflux of food into the nasal cavities
(item 3). Next, the palatopharyngeal folds on each side of the pharynx are
pulled medially to approximate each other (item 4). The vocal cords of the
larynx are strongly approximated, and the larynx is pulled upward and
anteriorly by the neck muscles. The upper esophageal sphincter relaxes
(item 1), and peristaltic contractions of the pharynx (item 2) move the
bolus of food into the esophagus.
TMP14 pp. 797–798
19. B) The three secretagogues for gastric acid secretion are gastrin,
histamine, and acetylcholine. These secretagogues have a synergistic or
multiplicative action on acid secretion, which means that blocking the
action of one secretagogue will cause the other secretagogues to be less
effective. Hence, blocking the actions of histamine with an H2
antagonist
causes gastrin and acetylcholine to be less effective in stimulating acid
secretion.
TMP14 p. 812
20. E) Helicobacter pylori is endemic in many underprivileged areas of the
world. At least 75% of persons with peptic ulcers have been found to have
chronic infection of the terminal portions of the gastric mucosa and initial
portions of the duodenal mucosa, most often caused by the bacterium H.
pylori. Once this infection begins, it can last a lifetime unless it is eradicated
by antibacterial therapy.
TMP14 p. 835
21. B) Patients with a lactase deficiency cannot digest milk products that
contain lactose (milk sugar). The operons of gut bacteria quickly switch
over to lactose metabolism, which results in fermentation that produces
copious amounts of gas (a mixture of hydrogen, carbon dioxide, and
methane). This gas, in turn, may cause a range of abdominal symptoms
including stomach cramps, bloating, and flatulence. The gas is absorbed by
blood (especially in the colon) and exhaled from the lungs. Blood glucose
levels do not increase because lactose is not digested to glucose and
galactose in these patients.
TMP14 pp. 823, 838
22. A) The appearance of mass movements after meals is facilitated by
gastrocolic and duodenocolic reflexes. These reflexes result from distention
of the stomach and duodenum. They are greatly suppressed when the
extrinsic autonomic nerves to the colon have been removed; therefore, the
reflexes are likely transmied by way of the autonomic nervous system.
All the gut reflexes are named with the anatomical origin of the reflex as
the prefix followed by the name of the gut segment in which the outcome
of the reflex is observed. For example, the duodenocolic reflex begins in
the duodenum and ends in the colon. When the duodenum is distended,
nervous signals are transmied to the colon, which stimulates mass
movements. The enterogastric reflex occurs when signals originating in the
intestines inhibit gastric motility and gastric secretion. The
intestinointestinal reflex occurs when overdistention or injury to a bowel
segment signals the bowel to relax. The rectosphincteric reflex, also called
the defecation reflex, is initiated when feces enter the rectum and stimulates
the urge to defecate.
TMP14 pp. 805–806
23. D) About 94% of the bile salts are reabsorbed into the blood from the
small intestine, with about half of this by diffusion through the mucosa in
the early portions of the small intestine and the remainder by an active
transport process through the intestinal mucosa in the distal ileum.
TMP14 p. 819
24. E) Vitamin B12 is absorbed in the ileum; this absorption requires intrinsic
factor, which is a glycoprotein secreted by parietal cells in the stomach.
Binding of intrinsic factor to dietary vitamin B12 is necessary for
aachment to specific receptors located in the brush border of the ileum.
Atrophic gastritis is a type of autoimmune gastritis that is mainly confined
to the acid-secreting corpus mucosa. The gastritis is diffuse, and severe
atrophy eventually develops. Ileal resection is likely to cause diarrhea but
not constipation. A gastric ulcer is possible but relatively unlikely. GERD is
caused by gastric acid and bile reflux into the esophagus; mucosal damage
and epithelial cell transformation lead to Barre esophagus, which is a
forerunner to adenocarcinoma, a particularly lethal cancer.
TMP14 p. 834
25. B) GIP is the only gastrointestinal hormone released by all three major
foodstuffs (fats, proteins, and carbohydrates). The presence of fat and
protein in the small intestine stimulates the release of CCK, but
carbohydrates do not stimulate its release. The presence of protein in the
antrum of the stomach stimulates the release of gastrin, but fat and
carbohydrates do not stimulate its release. Fat has a minor effect to
stimulate the release of motilin and secretin, but neither hormone is
released by the presence of protein or carbohydrate in the gastrointestinal
tract.
TMP14 p. 792
26. D) GIP is released by the presence of fat, carbohydrate, or protein in the
gastrointestinal tract. GIP is a strong stimulator of insulin release and is
responsible for the observation that an oral glucose load releases more
insulin and is metabolized more rapidly than an equal amount of glucose
administered intravenously. Intravenously administered glucose does not
stimulate the release of GIP. Neither CCK nor VIP stimulates the release of
insulin. GIP does not stimulate glucagon release, and glucagon has the
opposite effect of insulin; that is, it would decrease the rate of glucose
clearance from the blood. VIP does not stimulate GIP release.
TMP14 p. 792
27. C) Achlorhydria means simply that the stomach fails to secrete
hydrochloric acid. This condition is diagnosed when the pH of the gastric
secretions fails to decrease below 4 after stimulation by pentagastrin.
When acid is not secreted, pepsin also usually is not secreted. Even when it
is, the lack of acid prevents it from functioning because pepsin requires an
acid medium for activity. Thus, protein digestion is impaired.
TMP14 pp. 812, 834
28. C) Achalasia is a condition in which the LES fails to relax during
swallowing. As a result, food swallowed into the esophagus fails to pass
from the esophagus into the stomach. Trace C shows a high, positive
pressure that fails to decrease after swallowing, which is indicative of
achalasia. Trace A shows a normal pressure tracing at the level of the LES,
reflecting typical receptive relaxation in response to the food bolus. Trace E
is similar to trace C, but the pressures are subatmospheric. Subatmospheric
pressures occur only in the esophagus where it passes through the chest
cavity.
TMP14 pp. 799, 833
29. D) Pepsinogen is the precursor of the enzyme pepsin. Pepsinogen is
secreted from the peptic or chief cells of the gastric gland (also called the
oxyntic gland). To be converted from the precursor form to the active form
(pepsin), pepsinogen must come in contact with hydrochloric acid or
pepsin itself. Pepsin is a proteolytic enzyme that digests collagen and other
types of connective tissue in meats.
TMP14 pp. 811–812
30. A) Cholecystokinin (CCK) is the only gastrointestinal hormone that
inhibits gastric emptying under physiological conditions. This inhibition of
gastric emptying keeps the stomach full for a prolonged time, which is one
reason why a breakfast containing fat and protein “sticks with you” beer
than breakfast meals containing mostly carbohydrates. CCK also has a
direct effect on the feeding centers of the brain to reduce further eating.
Although CCK is the only gastrointestinal hormone that inhibits gastric
emptying, all the gastrointestinal hormones except for gastrin are released
to some extent by the presence of fat in the intestine.
TMP14 p. 792
31. F) All these factors can inhibit gastric acid secretion under normal
physiological conditions. Gastric acid stimulates the release of
somatostatin (a paracrine factor), which has a direct effect on the parietal
cell to inhibit acid secretion, as well as an indirect effect mediated by
suppression of gastrin secretion. Secretin and GIP inhibit acid secretion
through a direct action on parietal cells as well as indirectly through
suppression of gastrin secretion. Enterogastrones are unidentified
substances released from the duodenum and jejunum that directly inhibit
acid secretion. When acid or hypertonic solutions enter the duodenum, a
neurally mediated decrease in gastric acid secretion follows.
TMP14 p. 814
32. B) Gastrin stimulates gastric acid secretion, and secretin and GIP inhibit
gastric acid secretion under normal physiological conditions. It is
important to differentiate the physiological effects of the gastrointestinal
hormones from their pharmacologic actions. For example, gastrin and
CCK have identical actions on gastrointestinal function when large
pharmacologic doses are administered, but they do not share any actions
at normal physiological concentrations. Likewise, GIP and secretin share
multiple actions when pharmacologic doses are administered, but only one
action is shared at physiological concentrations: inhibition of gastric acid
secretion.
TMP14 p. 792
33. E) The cephalic phase of gastric secretion occurs before food enters the
stomach. Seeing, smelling, chewing, and anticipating food are perceived
by the brain, which “tells” the stomach to prepare for a meal. Stimuli for
the cephalic phase thus include mechanoreceptors in the mouth,
chemoreceptors (smell and taste), thought of food, and hypoglycemia.
Because the cephalic phase of gastric secretion is mediated entirely by way
of the vagus nerves, vagotomy can abolish the response. Antacids
neutralize gastric acid, but they do not inhibit gastric secretion. An
antigastrin antibody would aenuate (but not abolish) the cephalic phase
because this would have no direct effect on histamine and acetylcholine
stimulation of acid secretion. Atropine would aenuate the cephalic phase
by blocking acetylcholine receptors on parietal cells; however, atropine
does not abolish acetylcholine stimulation of gastrin secretion. A histamine
H2
blocker would aenuate the cephalic phase of gastric secretion but
would not abolish it.
TMP14 p. 813
34. C) MMCs (sometimes called interdigestive myoelectric complexes) are
peristaltic waves of contraction that begin in the stomach and slowly
migrate in an aboral direction along the entire small intestine to the colon.
By sweeping undigested food residue from the stomach, through the small
intestine, and into the colon, MMCs function to maintain low bacterial
counts in the upper intestine. Bacterial overgrowth syndrome can occur
when the normally low bacterial colonization in the upper gastrointestinal
tract increases significantly. It should be clear that an absence of MMCs
would decrease duodenal motility and gastric emptying. MMCs do not
have a direct effect on mass movements and swallowing.
TMP14 pp. 792–793
35. C) Trace C shows a basal subatmospheric pressure with a positive
pressure wave caused by passage of the food bolus. Trace A does not
correspond to any normal event in the esophagus. Trace B could represent
the LES in a patient with achalasia. Trace D depicts normal operation of
the LES. Trace E shows a basal positive pressure trace, which does not
occur where the esophagus passes through the chest cavity.
TMP14 p. 799
36. C) Gastric emptying is accomplished by coordinated activities of the
stomach, pylorus, and small intestine. Conditions that favor gastric
emptying include (a) increased tone of the orad stomach, which helps to
push chyme toward the pylorus; (b) forceful peristaltic contractions in the
stomach that move chyme toward the pylorus; (c) relaxation of the
pylorus, which allows chyme to pass into the duodenum; and (d) absence
of segmentation contractions in the intestine, which can otherwise impede
the entry of chyme into the intestine.
TMP14 pp. 800–801
37. A) The toxin from V. cholerae (cholera toxin) causes an irreversible increase
in cAMP levels (not cGMP levels) in the enterocytes located in the crypts of
Lieberkühn of the small intestine. This increase in cAMP causes an
irreversible opening of chloride channels on the luminal membrane.
Movement of chloride ions into the gut lumen causes a secondary
movement of sodium ions to maintain electrical neutrality. Water follows
the osmotic gradient created by sodium and chloride, causing a
tremendous increase in fluid loss into the gut lumen. Severe diarrhea
follows.
TMP14 pp. 830, 836
38. B) Cholera toxin causes an irreversible opening of chloride channels in the
enterocytes located in the crypts of Lieberkühn of the small intestine, as
indicated in the explanation for the previous answer. Although sodium
ions enter the gut lumen to maintain electrical neutrality after the flux of
chloride ions into the gut lumen, the sodium ions move through relatively
large paracellular pathways rather than through actual sodium channels.
Calcium, potassium, and magnesium do not have a significant role during
an infection with V. cholerae.
TMP14 pp. 830, 836
39. B) Enterocytes are derived from stem cells located in the crypts of
Lieberkühn of the small intestine. They mature as they migrate upward
toward the villus tip, where they are extruded into the gut lumen,
becoming part of the ingesta. In humans, the entire population of epithelial
cells is replaced in 3 to 6 days. Cholera also usually runs its course in 3 to 6
days. Because cholera toxin causes an irreversible opening of chloride
channels in the enterocytes, it is thought that the time course of cholera is
dictated by the life span of the enterocytes.
TMP14 pp. 830, 836
40. E) Cholecystokinin (CCK) is the only gastrointestinal hormone that
inhibits gastric emptying under normal physiological conditions. CCK
inhibits gastric emptying by relaxing the orad stomach, which increases its
compliance. When the compliance of the stomach is increased, the stomach
can hold a larger volume of food without excess buildup of pressure in the
lumen. None of the gastrointestinal hormones increases gastric emptying
under physiological conditions; however, gastrin, secretin, and GIP can
inhibit gastric emptying when pharmacologic doses are administered
experimentally.
TMP14 p. 792
41. E) The figure shows the time course of gastric pH, rate of acid secretion,
and stomach volume immediately before and for 4 hours after a meal.
Note that the pH of the gastric juice is lowest immediately before the meal
(not an answer choice) and 4 hours after consuming the meal (the correct
answer). It is a common misconception that the pH of the gastric juice is
lowest (most acidic) after a meal, when acid secretion is highest.
TMP14 pp. 813–814
42. A) Gastrin and CCK do not share any effects on gastrointestinal function
at normal physiological conditions; however, they have identical actions
on gastrointestinal function when pharmacologic doses are administered.
Gastrin stimulates gastric acid secretion and mucosal growth throughout
the stomach and intestines under physiological conditions. CCK stimulates
growth of the exocrine pancreas and inhibits gastric emptying under
normal conditions. CCK also stimulates gallbladder contraction, relaxation
of the sphincter of Oddi, and secretion of bicarbonate and enzymes from
the exocrine pancreas.
TMP14 p. 792
43. A) The frequency of slow waves is fixed in various parts of the gut. The
maximum frequency of smooth muscle contractions cannot exceed the
slow-wave frequency. The slow-wave frequency averages about 3 per
minute in the stomach, 12 per minute in the duodenum, 10 per minute in
the jejunum, and 8 per minute in the ileum. Therefore, the duodenum is
most likely to have the highest frequency of smooth muscle contractions.
TMP14 pp. 787–788
44. D) When feces enter the rectum, distention of the rectal wall initiates
signals that spread through the myenteric plexus to initiate peristaltic
waves in the descending colon, sigmoid colon, and rectum, all of which
force feces toward the anus. At the same time, the internal anal sphincter
relaxes, allowing the feces to pass. In people with transected spinal cords,
the defecation reflexes can cause automatic emptying of the bowel because
the external anal sphincter is normally controlled by the conscious brain
through signals transmied in the spinal cord.
TMP14 pp. 805–806
45. B) The use of NSAIDs may result in NSAID-associated gastritis or peptic
ulceration. Chronic gastritis, by definition, is a histopathologic entity
characterized by chronic inflammation of the stomach mucosa. When
inflammation affects the gastric corpus, parietal cells are inhibited, leading
to reduced acid secretion. Although diagnosis of chronic gastritis can only
be ascertained histologically, the administration of pentagastrin should
produce a less than expected increase in gastric acid secretion. Pentagastrin
is a synthetic gastrin composed of the terminal four amino acids of natural
gastrin plus the amino acid alanine. It has all the same physiological
properties of natural gastrin. Although gastrin and pentagastrin can both
stimulate growth of the duodenal mucosa, it should be clear that
intravenous pentagastrin would not cause substantial growth in the
context of a clinical test. In any case, chronic administration of pentagastrin
would not lead to a less than expected growth of the duodenal mucosa.
Pentagastrin is not expected to increase gastrin secretion, pancreatic
enzyme secretion, or pancreatic growth.
TMP14 pp. 833–834
46. E) The presence of acid, fay acids, and hyperosmotic solutions in the
duodenum and jejunum leads to suppression of acid secretion through a
variety of mechanisms. Acid stimulates the secretion of secretin from the
small intestine, which in turn inhibits acid secretion from parietal cells.
Acidification of the antrum and oxyntic gland area of the stomach
stimulates the release of somatostatin, which in turn inhibits acid secretion
by a direct action on the parietal cells and an indirect action mediated by
suppression of gastrin secretion. The presence of fay acids in the small
intestine stimulates the release of GIP, which inhibits acid secretion both
directly (parietal cell inhibition) and indirectly (by decreasing gastrin
secretion). Hyperosmotic solutions in the small intestine cause the release
of unidentified enterogastrones, which directly inhibit acid secretion from
parietal cells. Isotonic solutions have no effect on acid secretion.
TMP14 p. 814
47. C) Before a meal, when the stomach is empty, the pH of the gastric juice is
at its lowest point, and acid secretion is suppressed. Acid secretion is
suppressed in part because (a) the concentrated hydrogen ions in the
gastric juice stimulate somatostatin release, which has a direct action to
decrease the secretion of both gastrin and acid, and (b) the acid itself has a
direct effect to suppress parietal cell secretions. When a meal is taken, the
buffering effects of the food cause the gastric pH to increase, which in turn
decreases somatostatin release. Cholecystokinin and vasoactive intestinal
peptide do not have a role in the regulation of gastric acid secretion.
TMP14 p. 814
48. D) The act of vomiting is preceded by antiperistalsis that may begin as far
down in the gastrointestinal tract as the ileum. Distention of the upper
portions of the gastrointestinal tract (especially the duodenum) becomes
the exciting factor that initiates the actual act of vomiting. At the onset of
vomiting, strong contractions occur in the duodenum and stomach along
with partial relaxation of the lower esophageal sphincter. From then on, a
specific vomiting act ensues that involves (a) a deep breath, (b) relaxation
of the upper esophageal sphincter, (c) closure of the glois, and (d) strong
contractions of the abdominal muscles and diaphragm.
TMP14 pp. 837–838
49. E) Secretin inhibits gastrin secretion from normal G-cells in the antrum
and duodenum but stimulates gastrin secretion in gastrinoma cells. Any
increase in serum gastrin concentration greater than 110 pg/ml above
baseline after administration of human secretin is diagnostic of gastrinoma
(also called Zollinger-Ellison syndrome). The secretin test is considered the
most sensitive and accurate diagnostic method for gastrinoma.
TMP14 pp. 792, 814
50. E) Essentially all proteolytic enzymes are secreted in an inactive form,
which prevents autodigestion of the secreting organ. Enterokinase is
physically aached to the brush border of the enterocytes that line the
inner surface of the small intestine. Enterokinase activates trypsinogen to
become trypsin in the gut lumen. The trypsin then catalyzes the formation
of additional trypsin from trypsinogen, as well as several other
proenzymes (e.g., chymotrypsinogen, procarboxypeptidase, proelastase).
Pepsin is first secreted as pepsinogen, which has no proteolytic activity.
However, as soon as it comes into contact with hydrochloric acid, and
especially in contact with previously formed pepsin plus hydrochloride
acid, it is activated to form pepsin.
TMP14 p. 815
51. B) Neither plasma gastrin levels nor the rate of acid secretion is diagnostic
for duodenal ulcer. However, when patients with a duodenal ulcer are
pooled together, they exhibit a statistically significant increase in the rate
of acid secretion and a statistically significant decrease in plasma gastrin
levels. How is this possible? The basal and maximal acid secretion rates of
normal subjects range from 1 to 5 mEq/h and from 6 to 40 mEq/h,
respectively, which overlaps with the basal (2-10 mEq/h) and maximal (30-
80 mEq/h) acid secretion rates of persons with a duodenal ulcer. The
increase in acid secretion of the average person with a duodenal ulcer
suppresses the secretion of gastrin from the antrum of the stomach. It
should be obvious that endoscopy is diagnostic for duodenal ulcer.
TMP14 pp. 834–835
52. E) All of the gastrointestinal hormones are released after a meal, and all
have physiological effects.
TMP14 p. 792
53. A) H. pylori is a bacterium that accounts for 95% of patients with a
duodenal ulcer and virtually 100% of patients with a gastric ulcer when
chronic use of aspirin or other NSAIDs are eliminated. H. pylori is
characterized by high urease activity, which metabolizes urea to NH3
(ammonia). Ammonia reacts with H+
to become ammonium (NH4
+
). This
reaction allows the bacterium to withstand the acid environment of the
stomach. The ammonium production is believed to be the major cause of
cytotoxicity because the ammonium directly damages epithelial cells,
increasing the permeability of the gastric mucosal barrier. Bile salts and
NSAIDs can also damage the gastric mucosal barrier, but these substances
are not directly related to H. pylori infection. Pepsin can exacerbate the
mucosal lesions caused by H. pylori infection, but pepsin levels are not
increased by H. pylori. It should be clear that gastrin does not mediate the
mucosal damage caused by H. pylori.
TMP14 p. 835
54. B) Slow-wave frequency is not affected significantly by either the
autonomic nervous system or hormones; it is relatively constant at any
given location in the small intestine. When a slow wave reaches a
threshold value, a calcium spike potential (action potential) occurs, and
calcium ions enter the smooth muscle cell, which causes it to contract.
Norepinephrine hyperpolarizes smooth muscle cells in the intestine and
thereby decreases the likelihood that the membrane potential can reach a
threshold value. Therefore, norepinephrine does not affect the basal slowwave frequency of 10 occurrences per minute but does lower the
contraction frequency of the smooth muscle cells to zero occurrences per
minute in this problem.
TMP14 pp. 788, 802–803
55. E) The defecation reflex (also called the rectosphincteric reflex) occurs when
feces enter the rectum. When the rectum is stretched, the internal anal
sphincter relaxes, and the rectum contracts, pushing the feces toward the
anus. The external anal sphincter is controlled voluntarily and can be
contracted when defecation is not possible. Therefore, when a person feels
the urge to defecate, the internal anal sphincters are relaxed, the rectum
contracts, and the external anal sphincter is either contracted or relaxed
depending on the circumstances.
TMP14 pp. 805–806
56. E) Oxygen is shunted from the artery of a villus into its venous drainage
so that by the time the arterial blood reaches the villus tip, the oxygen
tension has been reduced to about 10 mm Hg. Adenosine dilates the villus
artery, increasing blood flow to the villus tip. This increase in blood flow
decreases the residence time for blood in the artery so that greater amounts
of oxygen can reach the villus tip, thus increasing the oxygen tension at the
villus tip. Factors that decrease intestinal blood flow (e.g., hemorrhagic
shock and a severe degree of exercise) can lead to ischemic death of villi
because of their low level of oxygenation.
TMP14 pp. 795–796
57. E) One of the most critical actions of gastrointestinal hormones is their
trophic activity. Gastrin can stimulate mucosal growth throughout the
gastrointestinal tract as well as growth of the exocrine pancreas. If most of
the endogenous gastrin is removed by antrectomy, the gastrointestinal
tract atrophies. Exogenous gastrin prevents the atrophy. Partial resection
of the small intestine for tumor removal, morbid obesity, or other reasons
results in hypertrophy of the remaining mucosa. The mechanism for this
adaptive response is poorly understood. Both cholecystokinin and secretin
stimulate growth of the exocrine pancreas. GIP and motilin do not appear
to have trophic actions on the gastrointestinal tract.
TMP14 p. 792
58. B) Relaxation of the ileocecal sphincter occurs with or shortly after eating.
This reflex has been termed the gastroileal reflex. It is not clear whether the
reflex is mediated by gastrointestinal hormones (gastrin and
cholecystokinin) or extrinsic autonomic nerves to the intestine. Note that
the gastroileal reflex is named with the origin of the reflex first (gastro) and
the target of the reflex named second (ileal). This method of naming is
characteristic of all the gastrointestinal reflexes. The enterogastric reflex
involves signals from the colon and small intestine that inhibit gastric
motility and gastric secretion. The gastrocolic reflex causes the colon to
evacuate when the stomach is stretched. The intestino-intestinal reflex
causes a bowel segment to relax when it is overstretched. The
rectosphincteric reflex is also called the defecation reflex.
TMP14 p. 803
59. D) Damage to the gastric mucosal barrier allows hydrogen ions to backleak into the mucosa in exchange for sodium ions. A low pH in the mucosa
causes mast cells to leak histamine, which damages the vasculature,
causing ischemia. The ischemic mucosa allows a greater leakage of
hydrogen ions—leading to more cell injury and death—resulting in a
vicious cycle. Factors that normally strengthen the gastric mucosal barrier
include mucus (which impedes the influx of hydrogen ions), gastrin
(which stimulates mucosal growth), certain prostaglandins (which can
stimulate mucus secretion), and various growth factors that can stimulate
growth of blood vessels, gastric mucosa, and other tissues. Factors that
weaken the gastric mucosal barrier include H. pylori (a bacterium that
produces toxic levels of ammonium), as well as aspirin, NSAIDs, ethanol,
and bile salts.
TMP14 pp. 833–834
60. B) Fat entering the small intestine is first emulsified into smaller globules
by bile released from the gallbladder. Pancreatic lipase in conjunction with
the co-enzyme colipase then digests the fat (which is mostly triglycerides)
into monoglycerides and free fay acids; these substances then become
surrounded by bile salts to form water-soluble aggregates called micelles.
When a micelle makes contact with an enterocyte of the intestinal wall, the
monoglycerides and free fay acids diffuse directly through the cell
membrane into the enterocyte; triglycerides are too large to be absorbed.
Once inside the enterocyte, the monoglycerides and free fay acids form
new triglyceride molecules that are subsequently packaged by the Golgi
apparatus into chylomicrons. The chylomicrons exocytose at the
basolateral membrane of the enterocyte and enter a lymphatic capillary
(central lacteal) in the villus.
TMP14 p. 826
61. C) The medical treatment of gastric ulcers is aimed at restoring the
balance between acid secretion and mucosal protective factors. Proton
pump inhibitors are drugs that covalently bind and irreversibly inhibit the
H+
/K+
adenosine triphosphatase (ATPase) pump, effectively inhibiting acid
release. Therapy can also be directed toward histamine release, that is, H2
blockers, such as cimetidine (Tagamet), ranitidine (Zantac), famotidine
(Pepcid), and nizatidine (Axid). These agents selectively block the H2
receptors in the parietal cells. Antibiotic therapy is used to eradicate the H.
pylori infection. NSAIDs can cause damage to the gastric mucosal barrier,
which is a forerunner of gastric ulcer.
TMP14 p. 835
62. B) Movement of chloride ions out of cells leads to secretion of fluid by
cells. CF is caused by abnormal chloride ion transport on the apical surface
of epithelial cells in exocrine gland tissues. The CF transmembrane
regulator (CFTR) protein functions both as a cAMP-regulated Cl − channel
and, as its name implies, a regulator of other ion channels. The fully
processed form of CFTR is found in the plasma membrane of normal
epithelia. Absence of CFTR at appropriate cellular sites is often part of the
pathophysiology of CF. However, other mutations in the CF gene produce
CFTR proteins that are fully processed but are nonfunctional or only
partially functional at the appropriate cellular sites.
TMP14 p. 830
63. A) Hydrogen ions leak into the mucosa when it is damaged. As the
hydrogen ions accumulate in the mucosa, the intracellular buffers become
saturated, and the pH of the cells decreases, resulting in injury and cell
death. The hydrogen ions also damage mast cells, causing them to secrete
excess amounts of histamine. The histamine exacerbates the condition by
damaging blood capillaries within the mucosa. The result is focal ischemia,
hypoxia, and vascular stasis. The mucosal lesion is a forerunner of gastric
ulcer. Mucus secretion helps strengthen the gastric mucosal barrier
because mucus impedes the leakage of hydrogen ions into the mucosa.
Various proton pump inhibitors are used as a treatment modality for
gastric ulcers because these inhibitors can decrease the secretion of
hydrogen ions (protons). The tight junctions between cells within the
mucosa help prevent the back-leak of hydrogen ions. Vagotomy was once
used to treat gastric ulcer disease because severing or crushing the vagus
nerve decreases gastric acid secretion.
TMP14 pp. 833–834
64. D) The fundus of the stomach and lower esophageal sphincter both relax
during a swallow while the bolus of food is still higher in the esophagus.
This phenomenon is called receptive relaxation. Receptive relaxation is
mediated by afferent and efferent pathways in the vagus nerves. Nitric
oxide is the neurotransmier thought to mediate receptive relaxation at the
smooth muscle cell. Motilin is a gastrointestinal hormone that mediates
migrating motility complexes (also called housekeeping contractions); these
contractions occur between meals. Gastrin and histamine do not have
significant effects on smooth muscle contraction or relaxation at
physiological levels. Norepinephrine can decrease smooth muscle
contraction in the small intestine but is not involved in receptive
relaxation.
TMP14 p. 799
65. A) Mass movements force feces into the rectum. When the walls of the
rectum are stretched by the feces, the defecation reflex is initiated, and a
bowel movement follows when this is convenient. Mass movements do not
affect gastric motility. Haustrations are bulges in the large intestine caused
by contraction of adjacent circular and longitudinal smooth muscle. It
should be clear that mass movements in the colon do not affect esophageal
contractions or pharyngeal peristalsis.
TMP14 pp. 805–806
66. E) This woman has gastroparesis (also called delayed gastric emptying). This
disorder slows or at times even stops the movement of chyme from the
stomach to the duodenum. Diabetes is the most commonly known cause of
gastroparesis; it occurs in about 20% of persons with type 1 diabetes. The
high blood glucose level is thought to damage the vagus nerves and
thereby delay gastric emptying.
TMP14 pp. 800–801
67. C) Acid acts directly on somatostatin cells to stimulate the release of
somatostatin. The somatostatin decreases acid secretion by directly
inhibiting the acid-secreting parietal cells and indirectly by inhibiting
gastrin secretion from G-cells in the antrum. Acid is a weak stimulus for
CCK release, but CCK does not inhibit (or stimulate) gastrin release. Acid
does not stimulate GIP release. Fay acids are a weak stimulus for motilin,
but motilin does not affect gastrin release. Fay acids are not thought to
stimulate somatostatin release.
TMP14 pp. 792, 814
68. C) All five gastrointestinal hormones are released from both the
duodenum and jejunum. Only gastrin is released from the antrum. Small
amounts of cholecystokinin and secretin are also released from the ileum.
No gastrointestinal hormones are released from the colon or esophagus.
TMP14 p. 792
69. D) Primary peristalsis of the esophagus is a continuation of pharyngeal
peristalsis; central control originates in the swallowing center located in
the medulla and pons. Visceral somatic fibers in the vagus nerves directly
innervate smooth muscle fibers of the pharynx and upper esophagus,
which coordinate pharyngeal peristalsis and primary peristalsis of the
esophagus. Esophageal contractions can occur independently of vagal
stimulation by a local stretch reflex initiated by the food bolus itself; this
phenomenon is called secondary peristalsis. Although the vagus nerves can
stimulate gastric acid secretion, gastrin release, and pancreatic bicarbonate
secretion, these processes can be activated by other mechanisms. Thus,
elimination of vagal stimulation does not completely eliminate them.
TMP14 p. 799
70. D) Persons with duodenal ulcers have about 2 billion parietal cells and
can secrete about 40 mEq H+
per hour. Unaffected individuals have about
50% of these values. Plasma gastrin levels are related inversely to acid
secretory capacity because of a feedback mechanism by which antral
acidification inhibits gastrin release. Thus, plasma gastrin levels are
usually reduced in persons with duodenal ulcers. Maximal acid secretion
and plasma gastrin levels are not diagnostic for duodenal ulcer disease
because of significant overlap with the normal population among persons
in each group.
TMP14 pp. 834–835
71. A) Intrinsic factor is a glycoprotein secreted from parietal cells (i.e., acidsecreting cells in the stomach) that is necessary for absorption of vitamin
B12. The patient has a diminished capacity to secrete acid because of
chronic gastritis. Because acid and intrinsic factor are both secreted by
parietal cells, a diminished capacity to secrete acid is usually associated
with diminished capacity to secrete intrinsic factor. Ptyalin, also known as
salivary amylase, is an enzyme that begins carbohydrate digestion in the
mouth. The secretion of ptyalin is not affected by gastritis. Rennin, known
also as chymosin, is a proteolytic enzyme synthesized by chief cells in the
stomach. Its role in digestion is to curdle or coagulate milk in the stomach,
a process of considerable importance in very young animals. It should be
clear that saliva secretion is not affected by gastritis. Trypsin is a
proteolytic enzyme secreted by the pancreas.
TMP14 p. 834
72. C) Parietal cells have receptors for all four substances shown. Stimulation
of receptors for gastrin, acetylcholine, and histamine lead to increased
secretion of gastric acid; stimulation of somatostatin receptors inhibits
gastric acid secretion.
TMP14 p. 813
73. C) This woman has celiac disease, also called gluten-sensitive
enteropathy, which is a chronic disease of the digestive tract that interferes
with the absorption of nutrients from food. Mucosal lesions seen on upper
gastrointestinal biopsy specimens are the result of an abnormal, genetically
determined, cell-mediated immune response to gliadin, a constituent of
the gluten found in wheat; a similar response occurs to comparable
proteins found in rye and barley. Gluten is not found in oats, rice, or corn.
When persons with celiac disease ingest gluten, the mucosa of their small
intestine is damaged by an immunologically mediated inflammatory
response, which results in malabsorption and maldigestion at the brush
border. Digestion of fat is normal in persons with celiac disease because
lipase secreted by the pancreas still functions normally. Malabsorption in
celiac disease increases the stool content of carbohydrates, fat, and
nitrogen. There is no cure for celiac disease, but a strict gluten-free diet can
help manage symptoms and promote intestinal healing.
TMP14 pp. 835–836
74. B) This infant has Hirschsprung’s disease, which is characterized by a
congenital absence of ganglion cells in the distal colon, resulting in a
functional obstruction. Prolonged fecal stasis can lead to enterocolitis (i.e.,
inflammation of the colon); full-thickness necrosis and perforation can
occur in severe cases. In achalasia, the lower esophageal sphincter fails to
relax during swallowing. Halitosis (bad breath) can occur in persons with
Hirschsprung’s disease, but this condition is not serious. Peptic ulcer and
pancreatitis (inflammation of the pancreas) are not common in persons
with Hirschsprung’s disease.
TMP14 p. 836
75. C) About 20% of persons older than 65 years have gallstones
(cholelithiasis) in the United States, and 1 million newly diagnosed cases of
gallstones are reported each year. Gallstones are the most common cause
of biliary obstruction. Regardless of the cause of gallstones, serum
bilirubin values (especially direct or conjugated) are usually elevated.
Indirect or unconjugated bilirubin values are usually normal or only
slightly elevated. Only choice C shows a high level of direct bili rubin
(conjugated bilirubin) compared with the level of indirect bilirubin
(unconjugated bilirubin).
TMP14 pp. 835, 874–875
76. A) Intestinal absorption of immunoglobulins (present in colostrum)
during early infancy occurs by endocytosis in the duodenum and jejunum.
This ability to absorb large molecules by endocytosis occurs during the
first several months of life but does not occur thereafter (except in the
ileum for absorption of vitamin B12). Facilitated diffusion, passive
diffusion, and primary and secondary active transport are all normal
transport processes in enterocytes.
TMP14 pp. 830–831
77. C) Pancreatitis is inflammation of the pancreas. The pancreas secretes
digestive enzymes into the small intestine that are essential in the digestion
of fats, proteins, and carbohydrates. Reduced secretion of fluid into the
pancreatic ducts in cystic fibrosis (CF) cause these digestive enzymes to
accumulate in the ducts. The digestive enzymes then become activated in
the pancreatic ducts (which typically would not occur) and can begin to
“digest” the pancreas, leading to inflammation and a myriad of other
problems (cysts and internal bleeding). Enterokinase is located at the brush
border of intestinal enterocytes, where it normally activates trypsin from
its precursor, trypsinogen. Trypsin inhibitor is normally present in the
pancreatic ducts, where it prevents trypsin from being activated and thus
prevents autodigestion of the pancreas. When the ducts are blocked in
cystic fibrosis, the available trypsin inhibitor is insufficient to prevent
trypsin from being activated. Excessive secretion of CCK does not occur in
persons with CF. Gallstone obstruction can lead to pancreatitis (by
autodigestion) when the obstruction prevents pancreatic juice from
entering the intestine, but this is unrelated to CF.
TMP14 pp. 815, 835
Unit XIII: Metabolism and
Temperature Regulation
1. A 54-year-old man eats a bowl of ice cream. Fifty minutes later,
chylomicrons enter his venous system through the thoracic duct.
Which of the following best describes the major constituent of a
typical chylomicron in this man?
A) Apoprotein B
B) Cholesterol
C) Monoglycerides
D) Phospholipids
E) Triglycerides
2. All the following tissues can use fay acids for energy EXCEPT
one. Which one is this EXCEPTION?
A) Brain
B) Heart
C) Kidney
D) Liver
E) Skeletal muscle
3. A 56-year-old woman with chronic liver disease is brought to the
physician because of confusion, a depressed level of consciousness,
personality changes, and intellectual impairment. Laboratory tests
support a diagnosis of liver encephalopathy. Which of the
following blood factors is the most likely cause of this condition?
A) Ammonia
B) Arginine
C) Citrulline
D) Ornithine
E) Urea
4. Elimination of bilirubin from the body requires several steps under
normal conditions that include (1) conjugation of bilirubin with
glucuronic acid, (2) carriage of bilirubin by albumin in plasma, (3)
excretion of bilirubin into bile, and (4) uptake of free bilirubin by
hepatocytes. Which of the following best describes the correct
temporal order of events?
A) 4, 3, 2, 1
B) 3, 1, 4, 2
C) 3, 4, 1, 2
D) 2, 1, 4, 3
E) 4, 2, 1, 3
F) 2, 4, 1, 3
G) 1, 2, 3, 4
H) 1, 3, 2, 4
5. A 45-year-old physician binge drinks alcohol three to five times per
week and does not eat during the alcoholic binges. Which of the
following long-term complications are possible in this man?
Jaundice Ascites Esophageal Varices Peripheral Edema
A) No Yes Yes No
B) No No Yes Yes
C) Yes Yes No Yes
D) Yes Yes Yes No
E) Yes Yes Yes Yes
6. A 54-year-old woman visits the physician because of upper
abdominal pain and vomiting. Physical examination shows upper
abdominal tenderness and diminished bowel sounds. Blood tests
show a 3-fold increase in plasma amylase levels compared with
normal. CT scan shows a mass at the papilla of Vater. Activation of
which of the following substances is the most likely cause of this
woman’s condition?
A) Chymotrypsin
B) Enterokinase
C) Lipase
D) Trypsin inhibitor
E) Trypsinogen
7. A 64-year-old, unacclimatized man works outside on a hot day.
Compared with an acclimatized person, this unacclimatized man is
most likely to have which of the following?
A) Decreased loss of sodium chloride in sweat only
B) Decreased sweat production only
C) Decreased sweat production and increased loss of sodium
chloride in sweat
D) Increased loss of sodium chloride in sweat only
E) Increased sweat production only
F) Increased sweat production and decreased loss of sodium
chloride in sweat
8. A 29-year-old man sits nude in a room that has a temperature of
71°F and relative humidity of 50%. He is 6 feet, 2 inches tall and
weighs 205 lb (body mass index = 26.3). The greatest amount of
body heat is most likely to be lost by which of the following
mechanisms?
A) Conduction to air
B) Conduction to objects
C) Evaporation
D) Radiation
9. The hypothalamic set-point temperature normally averages about
98.6°F. Which of the following factors can alter the set-point level
for core temperature control?
A) Skin temperature only
B) Pyrogens only
C) Thyroxin only
D) Skin temperature and pyrogens only
E) Skin temperature, pyrogens, and thyroxin
10. Which of the following describes the neurotransmier released
from neurons that innervate sweat glands?
A) Norepinephrine
B) Acetylcholine
C) Epinephrine
D) Dopamine
E) Glycine
11. A 23-year-old student gets lost in a snowstorm and does not have
proper clothing or supplies to make a fire. The body temperature
of the student decreases slowly over the next 18 hours. At which of
the following temperatures is the ability of the hypothalamus to
regulate body temperature completely lost (in °F)?
A) 95.0
B) 92.5
C) 90.0
D) 87.5
E) 85.0
12. Fay acid degradation in mitochondria produces which twocarbon substance?
A) Acetyl coenzyme A
B) Carnitine
C) Glycerol
D) Glycerol 3-phosphate
E) Oxaloacetic acid
13. The following events occurred during the course of a fever in a 12-
year-old boy: (1) cutaneous vasodilation and sweating; (2) a return
of the set-point temperature to normal; (3) an increase in the setpoint temperature to 103°F; and (4) shivering, chills, and cutaneous
vasoconstriction. Which of the following best describes the correct
temporal order of events during the fever in this boy?
A) 4, 3, 2, 1
B) 3, 4, 2, 1
C) 2, 1, 4, 3
D) 4, 2, 1, 3
E) 3, 4, 1, 2
F) 1, 2, 3, 4
G) 2, 3, 1, 4
H) 1, 3, 2, 4
14. A 72-year-old man with a 25-year history of alcoholism and liver
disease visits his physician because of sudden weight gain. One
year ago, the man had a body mass index (BMI) of 24.9 kg/m2
;
today his BMI is 28.5 kg/m2
. Physical examination shows +3 edema
in his feet and moderate ascites. Which condition is most likely to
have promoted the development of both ascites and peripheral
edema in this man?
A) Decreased capillary hydrostatic pressure
B) Decreased plasma colloid osmotic pressure
C) Increased capillary hydrostatic pressure
D) Increased plasma colloid osmotic pressure
15. A 24-year-old student goes hiking in the Mojave Desert during
spring break. The environmental temperature is 105°F, and the
relative humidity is 20%. Which option best describes the major
mechanism of heat loss in this student?
A) Conduction to air
B) Conduction to objects
j
C) Convection
D) Evaporation
E) Radiation
16. A 32-year-old student consumes a meal containing 10% fat, 50%
carbohydrate, and 40% protein. Four hours later, the metabolic rate
has increased by about 30%, even though the student is siing at
rest. Which substance is most likely to cause the greatest increase
in metabolic rate in this student 4 hours after consuming the meal?
A) Carbohydrate
B) Fat
C) Protein
17. A 90-year-old man is found siing in his yard, vomiting on a hot
summer day with the lawnmower running. The man is confused
and dizzy. He is admied to the hospital as an emergency patient.
His body temperature is 105°F, his heart rate is 110 beats/min, and
his skin turgor is poor. Which symptom is unlikely in this man?
A) Headache
B) Hot skin
C) Hypotension
D) Nausea
E) Sweating
18. A 43-year-old woman on a camping trip has underestimated the
cool evening temperatures, so she wraps herself in a thin sheet of
polyester film with a reflective surface (Mylar), also known as an
emergency blanket or space blanket. She feels warm immediately.
Which heat loss mechanism most likely accounts for the
effectiveness of this paper-thin, reflective Mylar blanket?
A) Conduction to air
B) Conduction to objects
C) Convection
D) Evaporation
E) Radiation
Questions 19 and 20
Refer to the following figure to answer the next two questions.
19. Abundant amounts of adenosine triphosphate (ATP) in the
cytoplasm of the cell inhibit which step in glycolysis?
A) Conversion of glucose to glucose-6-phosphate
B) Conversion of fructose-6-phosphate to fructose-1,6-
diphosphate
C) Conversion of 1,3-diphosphoglyceric acid to 3-
phosphoglyceric acid
D) Conversion of phosphoenolpyruvic acid to pyruvic acid
20. Abundant amounts of adenosine diphosphate (ADP) or adenosine
monophosphate (AMP) stimulate which step in glycolysis?
A) Conversion of glucose to glucose-6-phosphate
B) Conversion of fructose-6-phosphate to fructose-1,6-
diphosphate
C) Conversion of 1,3-diphosphoglyceric acid to 3-
phosphoglyceric acid
D) Conversion of phosphoenolpyruvic acid to pyruvic acid
21. A 44-year-old woman with hepatic cirrhosis comes to her
physician for a checkup. Physical examination shows ascites. The
woman’s prothrombin time has doubled since her last visit 3
months ago, and her hematocrit is now 30%. What is the most
likely cause of this low hematocrit?
A) Colon cancer
B) Esophageal varices
C) Jaundice
D) Acute pancreatitis
E) Scleral icterus
22. During resting conditions, about 75% of the blood flowing
through the liver is from the portal vein, and the remainder is from
the hepatic artery. Which option best describes the liver circulation
in terms of resistance, pressure, and flow?
Resistance Pressure Flow
A) High High High
B) High Low High
C) Low High Low
D) Low Low High
E) Low Low Low
23. A scuba diver explores an underwater lava flow where the water
temperature is 102°F. Which profile best describes the mechanisms
of heat loss that are effective in this man?
Evaporation Radiation Convection Conduction
A) No No No Yes
B) No No No No
C) Yes Yes No Yes
D) No Yes No Yes
E) Yes Yes Yes Yes
24. A 34-year-old African American man is admied to the hospital
because of steadily increasing intense pain in the upper right side
of the abdomen. He is nauseated and vomiting. His hematocrit is
30. Ultrasonography shows the presence of gallstones. Which of
the following is the most likely major composition of the gallstones
in this man?
A) Bile pigments
B) Calcium carbonate
C) Calcium oxalate
D) Cholesterol
25. Deamination means removal of the amino groups from the amino
acids. Which substance is produced when deamination occurs by
transamination?
A) Acetyl coenzyme A
B) Ammonia
C) Citrulline
D) Ornithine
E) α-Ketoglutaric acid
26. Most of the energy released from a glucose molecule occurs as a
result of which process?
A) Citric acid cycle
B) Glycogenesis
C) Glycogenolysis
D) Glycolysis
E) Oxidative phosphorylation
27. A 32-year-old woman visits her physician because of loss of
appetite, fatigue, nausea, and dizziness. Physical examination
shows thinning hair. Blood tests show a hematocrit of 32. The
woman began following a vegetarian diet suggested by a friend 1
year ago. The physician suspects a dietary deficiently of which
substance?
A) Alanine
B) Glycine
C) Lysine
D) Serine
E) Tyrosine
28. A 45-year-old man is admied to the emergency department after
he was found lying in the street in an inebriated state. He is
markedly pale with icteric conjunctivae and skin. His abdomen is
distended, and he has shifting dullness, indicating ascites. His liver
is enlarged about 5 cm below the right costal margin and tender.
His spleen cannot be palpated. He has bilateral grade 2 edema of
the legs and feet. Which values of direct and indirect bilirubin (in
milligrams per deciliter) are most likely to be present in this man’s
plasma?
Direct Indirect
A) 1.1 1.2
B) 1.7 5.4
C) 2.4 2.5
D) 5.2 1.8
E) 5.8 7.2
Questions 29–31
The diagram shows the effects of changing the set-point temperature of
the hypothalamic temperature controller. The red line indicates the body
temperature, and the blue line represents the hypothalamic set-point
temperature. Use the figure to answer the next three questions.
29. Which set of changes occurs at point W compared with point V?
Shivering Sweating Vasoconstriction Vasodilation
A) No No No No
B) No Yes No Yes
C) No Yes Yes No
D) Yes No No Yes
E) Yes No Yes No
F) Yes Yes Yes Yes
30. Which set of changes occurs at point Y compared with point V?
Shivering Sweating Vasoconstriction Vasodilation
A) No No No No
B) No Yes No Yes
C) No Yes Yes No
D) Yes No No Yes
E) Yes No Yes No
F) Yes Yes Yes Yes
31. Which set of changes occurs at point X compared with point V?
Shivering Sweating Vasoconstriction Vasodilation
A) No No No No
B) No Yes No Yes
C) No Yes Yes No
D) Yes No No Yes
E) Yes No Yes No
F) Yes Yes Yes Yes
32. Which of the following is the most abundant source of highenergy phosphate bonds in the cells?
A) ATP
B) Phosphocreatine
C) ADP
D) Creatine
E) Creatinine
33. A 76-year-old man is admied to the emergency department after
being found lying in a field on a hot summer day. His body
temperature is 106°F, his blood pressure is normal, and his heart
rate is 160 beats/min. Which set of changes is most likely to be
present in this man?
Sweating Hyperventilation Vasodilation of Skin
A) No No No
B) No Yes Yes
C) Yes No Yes
D) Yes Yes No
E) Yes Yes Yes
34. Most of the energy for strenuous exercise that lasts for more than 5
to 10 seconds but less than 1 to 2 minutes comes from what source?
A) ATP
B) Anaerobic glycolysis
C) Oxidation of carbohydrates
D) Oxidation of lactic acid
E) Conversion of lactic acid into pyruvic acid
35. Erythrocytes are constantly dying and being replaced. Heme from
the hemoglobin is converted to what substance before being
eliminated from the body?
A) Bilirubin
B) Cholesterol
C) Cholic acid
D) Globin
E) Glucuronic acid
36. A 32-year-old pregnant woman in her third trimester is admied
to the emergency department because of severe upper right
quadrant pain after eating a meal of fried catfish. Her blood
pressure is 130/84 mm Hg, heart rate is 105 beats/min, and
respirations are 30/min. Her body mass index before pregnancy
was 45 kg/m2
. Physical examination shows abdominal guarding
and diaphoresis. Her serum bilirubin levels and white blood cell
count are both normal. Which of the following best describes this
patient’s condition?
A) Cholelithiasis
B) Constipation
C) Hepatitis
D) Pancreatitis
E) Peritonitis
37. An experimental device containing hepatocytes is developed to
provide effective support for patients with hepatic failure pending
liver transplantation. Hepatocyte viability is best documented by
an increase in which function?
A) Lactate dehydrogenase uptake
B) Ethanol output
C) Albumin output
D) Glucuronic acid uptake
E) Oxygen output
F) Carbon dioxide uptake
38. The metabolic rate of a person is typically expressed in terms of
the rate of heat liberation that results from the chemical reactions
of the body. Which factors tend to increase or decrease a person’s
metabolic rate?
Growth Hormone Fever Sleep Malnutrition
A) Decrease Decrease Decrease Decrease
B) Decrease Increase Decrease Increase
C)
Increase Increase Increase Increase
D)
Increase Increase Decrease Increase
E)
Increase Increase Decrease Decrease
39. An 8-year-old girl is taken to the physician because of diarrhea
and a red scaly rash. Physical examination shows mild cerebellar
ataxia. She is suspected of having pellagra because of these chronic
symptoms. However, she appears to be ingesting adequate
amounts of niacin in her diet, which is rich in meat. A brother has a
similar problem. Urine studies show large amounts of free amino
acids. Which diagnosis is most likely?
A) Alkaptonuria
B) Beriberi
C) Hartnup’s disease
D) Scurvy
E) Stickler’s syndrome
40. In a person with type 1 diabetes who is not receiving insulin
therapy and who has a fasting blood glucose of 400 mg/100 ml,
what would the respiratory quotient likely be 2 hours after eating a
light meal containing 60% carbohydrates, 20% protein, and 20%
fat?
A) 0.5
B) 0.7
C) 0.9
D) 1.0
E) 1.2
41. A 3-year-old white boy is extremely obese (weight of 37.5 kg), and
his parents report that he has a voracious appetite. What is the
most likely cause of his hyperphagia and obesity?
A) A lesion or destruction of the lateral hypothalamus
B) Excessive stimulation of the ventromedial nuclei of the
hypothalamus
C) A mutation that produces nonfunctional melanocortin-4
receptor protein
D) Excessive stimulation of pro-opiomelanocortin (POMC)
neurons
E) Excessive secretion of leptin
F) A mutation that prevents neuropeptide Y (NPY) formation in
hypothalamic neurons
42. Deficiency of which of the following would cause “night
blindness” in humans?
A) Vitamin A
B) Vitamin B1
C) Vitamin B6
D) Vitamin B12
E) Vitamin C
F) Niacin
43. Which changes would be expected to stimulate hunger in a person
who has not eaten for 24 hours?
A) Increased NPY in the hypothalamus
B) Increased leptin secretion
C) Increased peptide YY (PYY) secretion
D) Decreased ghrelin secretion
E) Activation of hypothalamic POMC neurons
F) Increased cholecystokinin secretion
44. Which of the following would be most important in contributing
to satiety after eating a large meal containing carbohydrates (50%),
fat (40%), and protein (10%)?
A) Release of cholecystokinin by the duodenum
B) Decreased leptin secretion
C) Increased release of endorphins
D) Increased ghrelin release by the stomach
E) Decreased release of PYY by the intestine
45. Deficiency of which vitamin is most likely to cause impaired blood
cloing?
A) Vitamin A
B) Vitamin B6
C) Vitamin C
D) Vitamin D
E) Vitamin K
46. Deficiency of which vitamin is the main cause of beriberi?
A) Vitamin A
B) Thiamine (vitamin B1
)
C) Riboflavin (vitamin B2
)
D) Vitamin B12
E) Pyridoxine (vitamin B6
)
47. Which of the following would tend to decrease hunger?
A) Increased release of endorphins
B) Increased ghrelin release by the stomach
C) Increased release of PYY by the intestine
D) Increased release of NPY by the hypothalamus
E) Increased release of cortisol by the adrenals
48. Which substance might be most useful in stimulating appetite in a
patient with cancer who has anorexia or cachexia?
A) Leptin
B) α-Melanocyte-stimulating hormone
C) PYY
D) Melanocortin-4 receptor antagonist
E) Ghrelin antagonist
F) Neuropeptide YY antagonist
49. The first stage in using triglycerides for energy is hydrolysis of the
triglycerides to which substances?
A) Acetyl coenzyme A and glycerol
B) Cholesterol and fay acids
C) Glycerol 3-phosphate and cholesterol
D) Glycerol and fay acids
E) Phospholipids and glycerol
50. Urinary nitrogen excretion measured in a patient is 16.0 g in 24
hours. What is the approximate amount of protein breakdown in
this patient for 24 hours in grams?
A) 16
B) 18
C) 100
D) 110
E) 120
Answers
1. E) During digestion, most triglycerides are split into monoglycerides
and fay acids. Then, while passing through the intestinal epithelial
cells, the monoglycerides and fay acids are resynthesized into new
molecules of triglycerides that enter the lymph as minute, dispersed
droplets called chylomicrons.
TMP14 pp. 853–854
2. A) The first stage in using triglycerides for energy is their hydrolysis into
fay acids and glycerol. Then, both the fay acids and the glycerol are
transported in the blood to the active tissues, where they will be
oxidized to give energy. Almost all cells—with some exceptions, such as
brain tissue and red blood cells—can use fay acids for energy.
TMP14 p. 856
3. A) Essentially all urea formed in the human body is synthesized in the
liver from ammonia. In the absence of the liver or in persons with
serious liver disease, ammonia accumulates in the blood. This
accumulation of ammonia is extremely toxic, especially to the brain, and
can lead to a state called hepatic coma or liver encephalopathy.
TMP14 pp. 869, 874
4. F) Bilirubin is formed during the degradation of red blood cells by
macrophages and released into plasma where it combines with albumin
(item 2). The free bilirubin is then taken up by hepatocytes (item 4),
where most of it is conjugated with glucuronic acid (item 1). The
conjugated bilirubin is then excreted from the hepatocytes by an active
transport process into the bile canaliculi and then into the intestines
(item 3).
TMP14 pp. 874–875
5. E) When liver parenchymal cells are destroyed, they are replaced with
fibrous tissue that eventually contracts around the blood vessels,
thereby greatly impeding the flow of portal blood through the liver. This
disease process is known as cirrhosis of the liver; it is a common
consequence of chronic alcoholism. The blockage of blood flow through
the liver raises the blood pressure in the upstream splanchnic organs,
causing ascites fluid to collect in the abdomen and varices to develop in
the lower esophagus. Peripheral edema can result from a failure of
hepatocytes to produce normal amounts of albumin (low colloid
osmotic pressure). Jaundice (hyperbilirubinemia) develops because of
damage to hepatocytes.
TMP14 pp. 871–872
6. E) This woman has acute pancreatitis caused by blockage of the papilla
of Vater by one or more gallstones. When a gallstone blocks the papilla
of Vater, the main secretory duct from the pancreas and the common
bile duct are blocked. The pancreatic enzymes are then dammed up in
the ducts and acini of the pancreas. Eventually, so much trypsinogen
accumulates that it overcomes the trypsin inhibitor in the secretions and
a small quantity of trypsinogen becomes activated to form trypsin.
When this happens, the trypsin activates still more trypsinogen, as well
as chymotrypsinogen and carboxypeptidase, resulting in a vicious circle
until most of the proteolytic enzymes in the pancreatic ducts and acini
become activated. These enzymes rapidly digest large portions of the
pancreas, sometimes completely and permanently destroying the ability
of the pancreas to secrete digestive enzymes.
TMP14 p. 835
7. C) Although a normal, unacclimatized person seldom produces more
than about 1 l of sweat per hour, when this person is exposed to hot
weather for 1 to 6 weeks, he or she begins to sweat more profusely, often
increasing maximum sweat production to as much as 2 to 3 l/hr. Also
associated with acclimatization is a further decrease in the concentration
of sodium chloride in the sweat, which allows progressively beer
conservation of body salt. Most of this effect is caused by increased
secretion of aldosterone by the adrenocortical glands, which results
from a slight decrease in sodium chloride concentration in the
extracellular fluid and plasma. An unacclimatized person who sweats
profusely often loses 15 to 30 g of salt each day for the first few days.
After 4 to 6 weeks of acclimatization, the loss is usually only 3 to 5
g/day.
TMP14 p. 911
8. D) The various methods by which heat is lost from the skin to the
surroundings include radiation, evaporation, and conduction to air and
conduction to objects. In a nude person siing inside at normal room
temperature, about 60% of total heat loss is by radiation, 22% by
evaporation, 15% by conduction to air, and 3% by conduction to objects.
TMP14 pp. 902–903
9. D) The set-point temperature in the hypothalamus is determined mainly
by the degree of activity of the heat temperature receptors in the
anterior hypothalamic- preoptic area. However, temperature signals
from the peripheral areas of the body, especially from the skin and
certain deep body tissues (spinal cord and abdominal viscera), also
contribute slightly to body temperature regulation. Many proteins,
breakdown products of proteins, and certain other substances,
especially lipopolysaccharide toxins released from bacterial cell
membranes, can cause the set-point of the hypothalamic thermostat to
rise. Substances that cause this effect are called pyrogens.
TMP14 pp. 909–910
10. B) The primary secretion of sweat is an active secretory product of the
epithelial cells lining the coiled portion of the sweat gland. Sympatheticcholinergic nerve fibers ending on or near the glandular cells elicit the
secretion. These are postganglionic sympathetic fibers that release
acetylcholine and hence are called sympathetic-cholinergic fibers; they
innervate sweat glands and some blood vessels.
TMP14 p. 904
11. E) When the body temperature has fallen below about 85°F, the ability
of the hypothalamus to regulate temperature is lost; it is greatly
impaired even when the body temperature falls below about 94°F. Part
of the reason for this diminished temperature regulation is that the rate
of chemical heat production in each cell is depressed almost 2-fold for
each 10°F decrease in body temperature. Also, sleepiness develops (later
followed by coma), which depresses the activity of the central nervous
system heat control mechanisms and prevents shivering.
TMP14 p. 911
12. A) Fay acids are degraded in mitochondria by the progressive release
of two-carbon segments in the form of acetyl coenzyme A. This process
is known as the beta-oxidation process for degradation of fay acids.
TMP14 pp. 856–857
13. B) The typical series of events that occur during a fever are shown in
the figure on the right. When pyrogens raise the set-point temperature
above its normal value, the body activates heat conservation and heat
production mechanisms that include cutaneous vasoconstriction,
piloerection, epinephrine secretion, and shivering. Within several
minutes, the body temperature increases to the elevated set-point value
of 103°F in this example. If the factor that is causing the high
temperature is removed, the hypothalamic set-point temperature
returns to a normal value of about 98.6°F, which leads to activation of
heat-loss mechanisms such as sweating and cutaneous vasodilation. The
body temperature then returns to its basal level.
TMP14 p. 910
14. B) This man has cirrhosis of the liver. Fluid accumulates in the
abdomen (ascites) for two main reasons: (1) decreased plasma colloid
osmotic pressure (COP) and (2) increased capillary hydrostatic pressure
in the splanchnic organs. The decrease in plasma COP results from
decreased production of albumin by liver hepatocytes; albumin
accounts for nearly 80% of the plasma COP. The low plasma COP also
promotes edema formation in the periphery, especially the feet. Liver
parenchymal cells are damaged or destroyed in persons with cirrhosis of
the liver. The cells are replaced with fibrous tissue that eventually
contracts around the blood vessels, thereby greatly impeding the flow of
portal blood through the liver. This increase in vascular resistance leads
to an increase in portal vein pressure, which in turn raises the capillary
hydrostatic pressure of the splanchnic organs. There is no reason to
assume that capillary hydrostatic pressure is also increased above
normal in the feet of this man.
TMP14 pp. 401, 867, 871
15. D) Evaporation is the only mechanism of heat loss from the body when
the environmental temperature is greater than the body temperature.
Each gram of water that evaporates from the surface of the body causes
0.58 kcal of heat to be lost from the body. Even when a person is not
sweating, water still evaporates insensibly from the skin and lungs at a
rate of 450 to 600 ml/day, which amounts to about 12 to 16 kcal of heat
loss per hour. Radiation, convection, and conduction are mechanisms of
heat loss when the body temperature is greater than the environmental
temperature.
TMP14 pp. 903–904
16. C) The metabolic rate increases after a meal because of various
chemical reactions associated with digestion, absorption, and storage of
food; this phenomenon is known as the thermogenic effect of food. After
a meal containing mostly carbohydrates and fats, the metabolic rate
usually increases by about 4%. However, a high-protein meal often
increases the metabolic rate by as much as 30%; this effect can last from
3 to 12 hours after the meal and is called the specific dynamic action of
proteins. Clearly, assimilation of proteins requires far more energy
expenditure compared with fats and carbohydrates.
TMP14 pp. 897–898
17. E) This man has heatstroke. When the body temperature rises into the
range of 105°F to 108°F, heatstroke likely follows. Heat loss mechanisms
are overwhelmed by excessive metabolic production of heat and
excessive environmental heat. Heatstroke is usually accompanied by
dehydration (poor skin turgor is common), which can produce nausea,
vomiting, hypotension, and fainting or dizziness. Interestingly, the skin
is frequently dry because the anterior hypothalamic-preoptic area of the
brain that normally initiates sweating is often compromised by the
elevation in body temperature.
TMP14 p. 911
18. E) Most of the heat loss from the body occurs by radiation in the form
of infrared heat waves, which is a type of electromagnetic wave. Heat
waves radiate from all objects toward the body, and the body radiates
heat waves to all surrounding objects. The reflective surface of the Mylar
blanket prevents heat loss by reflecting infrared heat waves from the
body back to the body, which causes the body to warm. At room
temperature, 60% of the heat loss occurs by radiation, 22% by
evaporation, 15% by conduction to air, and 3% by conduction to objects.
Convection (i.e., air currents) can increase heat loss by removing the
unstirred layer of air close to the skin.
TMP14 p. 903
19. B) Continual release of energy from glucose when energy is not needed
by the cells would be an extremely wasteful process. Both ATP and ADP
control the rate of chemical reactions in the energy metabolism
sequence. When ATP is abundant within the cell, it helps control energy
metabolism by inhibiting the conversion of fructose-6-phosphate to
fructose-1,6-diphosphate. It does so by inhibiting the enzyme
phosphofructokinase.
TMP14 p. 849
20. B) Both ADP and AMP increase the activity of the enzyme
phosphofructokinase and increase the conversion of fructose-6-
phosphate to fructose-1,6-diphosphate.
TMP14 p. 849
21. B) Esophageal varices are extremely dilated submucosal veins in the
lower third of the esophagus. The submucosal veins have a normal
diameter of about 1 mm and can enlarge to 1 to 2 cm with prolonged
portal hypertension, which is common in persons with cirrhosis of the
liver. The presence of ascites indicates that the patient has portal
hypertension. The dilated esophageal veins often bleed and thus lower
the hematocrit. Although colon cancers can also bleed, there is no reason
to assume colon cancer in this woman. Pancreatitis can occur in persons
with chronic alcoholism, but there is no evidence for this condition, and
substantial bleeding is not common in persons with pancreatitis.
Jaundice and scleral icterus (i.e., yellowing of the sclera) are common in
persons with cirrhosis, but these conditions are unlikely to cause
significant bleeding.
TMP14 pp. 871–872
22. D) The liver has a high blood flow, low vascular resistance, and low
blood pressure. During resting conditions, about 27% of the cardiac
output flows through the liver, yet the pressure in the portal vein
leading into the liver averages only 9 mm Hg. This high flow and low
pressure indicate that the resistance to blood flow through the hepatic
sinusoids is normally very low.
TMP14 p. 871
23. B) None of the mechanisms of heat loss is effective when a person is
placed in water that has a temperature greater than body temperature.
Instead, the body will continue to gain heat until the body temperature
becomes equal to the water temperature.
TMP14 p. 903
24. A) This man has sickle cell disease, which is a hemolytic disease that
results in the premature destruction of red blood cells. Release of
hemoglobin from damaged red blood cells leads to high levels of
bilirubin in the blood plasma. This increase in bilirubin can lead to the
development of pigment stones in the gallbladder that are composed
primarily of bilirubin.
TMP14 pp. 446, 875
25. B) The degradation of amino acids occurs almost entirely in the liver,
and it begins with deamination, which occurs mainly by the following
transamination schema: The amino group from the amino acid is
transferred to α-ketoglutaric acid, which then becomes glutamic acid.
The glutamic acid then transfers the amino group to still other
substances or releases it in the form of ammonia. In the process of losing
the amino group, the glutamic acid again becomes α-ketoglutaric acid,
so that the cycle can repeat again and again.
TMP14 pp. 868–869
26. E) About 90% of the total ATP produced by glucose metabolism is
formed during oxidation of the hydrogen atoms released during the
early stages of glucose degradation. This process is called oxidative
phosphorylation. Only two ATP molecules are formed by glycolysis, and
another two are formed in the citric acid cycle. ATP is not formed by
glycogenesis or glycogenolysis.
TMP14 p. 848
27. C) Lysine is an essential amino acid, which means that it must be
included in the diet because the body cannot synthesize it. Alanine,
glycine, serine, and tyrosine can be synthesized by the body and are
therefore considered nonessential amino acids. This woman has a lysine
deficiency, which is common in poorly designed vegetarian diets;
symptoms include nausea, fatigue, dizziness, anemia, loss of appetite,
and thinning hair. Good dietary sources of lysine include eggs, meat,
beans, legumes, soy, dairy products, and certain fish (such as cod and
sardines). L-lysine is a building block for all proteins in the body.
TMP14 pp. 866, 868
28. D) This man has cirrhosis of the liver. In this condition, the rate of
bilirubin production is normal, and the free bilirubin still enters the liver
cells and becomes conjugated in the usual way. The conjugated bilirubin
(direct) is mostly returned to the blood, probably by rupture of
congested bile canaliculi, so that only small amounts enter the bile. The
result is elevated levels of conjugated (direct) bilirubin in the plasma,
with normal or near-normal levels of unconjugated (indirect) bilirubin.
TMP14 pp. 871, 874–875
29. E) When the hypothalamic set-point temperature is greater than the
body temperature, the person feels cold and exhibits responses that lead
to an elevation of body temperature. These responses include shivering
and vasoconstriction, as well as piloerection and epinephrine secretion.
Shivering increases heat production. The increase in epinephrine
secretion causes an immediate increase in the rate of cellular
metabolism, which is an effect called chemical thermogenesis.
Vasoconstriction of the skin blood vessels decreases heat loss through
the skin.
TMP14 p. 910
30. B) When the hypothalamic set-point temperature is lower than the
body temperature, the person feels hot and exhibits responses that cause
body temperature to decrease. These responses include sweating and
vasodilation. Sweating increases heat loss from the body by evaporation.
Vasodilation of skin blood vessels facilitates heat loss from the body by
increasing the skin blood flow.
TMP14 p. 910
31. A) When the hypothalamic set-point temperature is equal to the body
temperature, the body exhibits neither heat loss nor heat conservation
mechanisms, even when the body temperature is far above normal.
Therefore, the person does not feel hot even when the body temperature
is 104°F.
TMP14 p. 910
32. B) Phosphocreatine contains high-energy phosphate bonds and is three
to eight times as abundant as ATP or ADP in a cell. Creatine does not
contain high-energy phosphate bonds. Creatinine is a breakdown
product of creatine phosphate in muscle.
TMP14 p. 894
33. B) This patient has heatstroke. Patients with heatstroke commonly
exhibit tachypnea and hyperventilation caused by direct central nervous
system stimulation, acidosis, or hypoxia. The blood vessels in the skin
are vasodilated, and the skin is warm. Sweating ceases in patients with
true heatstroke, most likely because the high temperature itself causes
damage to the anterior hypothalamic-preoptic area. The nerve impulses
from this area are transmied in the autonomic pathways to the spinal
cord and then through sympathetic outflow to the skin to cause
sweating.
TMP14 p. 911
34. B) Most of the extra energy required for strenuous activity that lasts for
more than 5 to 10 seconds but less than 1 to 2 minutes is derived from
anaerobic glycolysis. Release of energy by glycolysis occurs much more
rapidly than oxidative release of energy, which is much too slow to
supply the needs of the muscle in the first few minutes of exercise. ATP
and phosphocreatine already present in the cells are rapidly depleted in
less than 5 to 10 seconds. After the muscle contraction is over, oxidative
metabolism is used to reconvert much of the accumulated lactic acid
into glucose; the remainder becomes pyruvic acid, which is degraded
and oxidized in the citric acid cycle.
TMP14 p. 894
35. A) Hemoglobin is metabolized by tissue macrophages (also called the
reticuloendothelial system). The hemoglobin is first split into globin and
heme, and the heme ring is opened to produce free iron and a straight
chain of four pyrrole nuclei, from which bilirubin will eventually be
formed. The free bilirubin is taken up by hepatic cells, and most of it is
conjugated with glucuronic acid; the conjugated bilirubin passes into the
bile canaliculi and then into the intestines.
TMP14 p. 875
36. A) Cholelithiasis is the presence of gallstones (choleliths) in the
gallbladder or bile ducts. This patient exhibits typical symptoms caused
by gallstones.
TMP14 p. 820
37. C) Hepatocytes produce essentially all the albumin normally present in
blood. Viable hepatocytes use oxygen and produce carbon dioxide.
Glucuronic acid produced by hepatocytes is used to conjugate bilirubin,
forming bilirubin glucuronide. Lactate dehydrogenase is an enzyme that
converts pyruvic acid to lactic acid under anaerobic conditions.
TMP14 pp. 317, 873–874
38. E) Growth hormone can increase the metabolic rate 15% to 20% as a
result of direct stimulation of cellular metabolism. Fever, regardless of
its cause, increases the chemical reactions of the body by an average of
about 120% for every 10°C rise in temperature. The metabolic rate
decreases 10% to 15% below normal during sleep. Prolonged
malnutrition can decrease the metabolic rate 20% to 30%, presumably
because of the paucity of food substances in the cells.
TMP14 p. 898
39. C) This child has Hartnup’s disease. This condition resembles pellagra
(because of the symptoms of diarrhea, dementia, and dermatitis) and
may be misdiagnosed as a nutritional deficiency of niacin. Hartnup’s
disease is an autosomal-recessive trait caused by a defective gene that
codes for a sodium-dependent and chloride-independent neutral amino
acid transporter expressed mainly in kidney and intestinal epithelium.
Poor epithelial transport of neutral amino acids (such as tryptophan)
leads to poor absorption of dietary amino acids, as well as excess amino
acid excretion in the urine. Tryptophan is a precursor of niacin; it is an
essential amino acid that must be included in the diet. Alkaptonuria,
also called “black urine disease,” is a genetic disorder of phenylalanine
and tyrosine metabolism. Beriberi is caused by a nutritional deficit in
thiamine. Scurvy results from a deficiency of vitamin C, which is
required for collagen synthesis. Stickler’s syndrome is a group of genetic
disorders that affect connective tissues; it is characterized by eye
problems, hearing loss, joint problems, and facial abnormalities.
TMP14 p. 889
40. B) Type 1 diabetes is characterized by a lack of insulin. In the absence
of adequate insulin, lile carbohydrate can be used by the body’s cells,
and the respiratory quotient remains near that for fat metabolism (0.70).
TMP14 p. 879
41. C) Mutations that produce a nonfunctional melanocortin-4 receptor
cause extreme obesity and may account for as much as 5% to 6% of early
onset, morbid obesity in children. All the other changes would tend to
reduce food intake and/or increase energy expenditure and thus cause
weight loss rather than obesity.
TMP14 pp. 880–881
42. A) One of the basic functions of vitamin A is in the formation of retinal
pigments and therefore the prevention of night blindness.
TMP14 p. 888
43. A) NPY is an orexigenic neurotransmier that stimulates feeding and is
increased during food deprivation. Leptin, PYY, cholecystokinin, and
activation of POMC neurons are all reduced by fasting. Ghrelin is
increased, not decreased, by fasting.
TMP14 pp. 880–883
44. A) Cholecystokinin is released mainly in response to fats and proteins
entering the duodenum and activates sensory receptors in the
duodenum, sending messages to the brain stem via vagal afferents that
contribute to satiation and meal cessation. All the other changes would
tend to increase rather than decrease food intake.
TMP14 p. 882
45. E) Vitamin K is an essential co-factor to a liver enzyme that adds a
carboxyl group to factors II (prothrombin), VII (proconvertin), IX, and X,
all of which are important to blood coagulation. The other vitamins
listed are not directly involved in coagulation.
TMP14 p. 891
46. B) Thiamine is needed for the final metabolism of carbohydrates and
amino acids. Decreased utilization of these nutrients secondary to
thiamine deficiency is responsible for many of the characteristics of
beriberi, including peripheral vasodilation and edema, lesions of the
central and peripheral nervous system, and gastrointestinal tract
disturbances.
TMP14 pp. 888–889
47. C) PYY is released from most parts of the intestinal tract, but especially
from the ileum and colon, in response to food intake. Increased levels of
PYY have been shown to decrease food intake. All the other changes
tend to increase food intake.
TMP14 pp. 880–881
48. D) Antagonists of melanocortin-4 receptors have been shown to
markedly aenuate anorexia (i.e., reduced food intake due to decreased
appetite) and cachexia (i.e., increased energy expenditure as well as
decreased food intake) by blocking hypothalamic melanocortin-4
receptors. All the other choices would tend to decrease appetite and/or
increase energy expenditure, exacerbating the anorexia or cachexia of a
patient with cancer.
TMP14 p. 880–881, 887
49. D) Triglycerides are hydrolyzed to glycerol and fay acids, which, in
turn, are oxidized to provide energy. Almost all cells, with the exception
of some brain tissue, can use fay acids almost interchangeably with
glucose for energy.
TMP14 pp. 856–857
50. D) The rate of protein metabolism can be estimated by measuring the
nitrogen in the urine, then adding 10 percent (about 90% of the nitrogen
in proteins is excreted in the urine) and multiplying by 6.25 (100/16)
because the average protein contains about 16% nitrogen.
TMP14 p. 879
Unit XIV: Endocrinology and
Reproduction
1. Which of the following is expected to exhibit the greatest biological
activity?
A) Insulin like growth factor-1 free in the plasma
B) Cholecalciferol (vitamin D3)
C) Cortisol bound to corticosteroid binding globulin
D) T4 bound to thyroxine binding globulin
E) Aldosterone bound to plasma albumin
2. Which receptor controls nitric oxide (NO) release to cause
vasodilation during penile erection?
A) Leptin receptor
B) Angiotensin AT1 receptor
C) Endothelin ETA receptor
D) Muscarinic receptor
3. After menopause, hormone replacement therapy with estrogen-like
compounds is effective in preventing the progression of
osteoporosis. What is the mechanism of their protective effect?
A) They stimulate the activity of osteoblasts
B) They increase absorption of calcium from the gastrointestinal
tract
C) They stimulate calcium reabsorption by the renal tubules
D) They stimulate parathyroid hormone (PTH) secretion by the
parathyroid gland
4. Neurons that secrete antidiuretic hormone or oxytocin terminate in
which of the following structures?
A) Posterior pituitary
B) Median eminence
C) Mammillary body
D) Paraventricular nucleus
E) Supraoptic nucleus
5. Which of the following represents a physiological action of growth
hormone?
A) Increases the breakdown of muscle protein
B) Increases utilization of glucose in muscle
C) Decreases storage of lipids in adipose cells
D) Decreases gene transcription
E) Decreases gluconeogenesis in the liver
6. Which hormones antagonize the effect of NO and cause the penis to
become flaccid after orgasm?
A) Endothelin and norepinephrine
B) Estrogen and progesterone
C) Luteinizing hormone (LH) and follicle-stimulating hormone
(FSH)
D) Progesterone and LH
Questions 7–9
The red lines in the above figure illustrate the normal relationships
between plasma insulin concentration and glucose production in the liver
and between plasma insulin concentration and glucose uptake in muscle.
Use this figure to answer Questions 7–9.
7. Which lines most likely illustrate these relationships in a patient
with type 2 diabetes?
A) A and C
B) A and D
C) B and C
D) B and D
8. Which lines most likely illustrate these relationships in a patient
with acromegaly?
A) A and C
B) A and D
C) B and C
D) B and D
9. Line D most likely illustrates the influence of which of the
following?
A) Exercise
B) Obesity
C) Growth hormone (GH)
D) Cortisol
E) Glucagon
10. Thecal cells in the follicle are not able to produce what sex steroid?
A) Estradiol
B) Testosterone
C) Progesterone
D) Dihydrotestosterone
11. A baby is born with a penis, a scrotum with no testes, no vagina,
and XX chromosomes. This condition is referred to as
hermaphroditism. What could cause this abnormality?
A) Abnormally high levels of human chorionic gonadotropin
(hCG) production by the trophoblast cells
B) The presence of a testosterone-secreting tumor in the mother’s
right adrenal gland
C) Abnormally high levels of LH in the maternal blood
D) Abnormally low levels of testosterone in the maternal blood
E) Abnormally low rates of estrogen production by the placenta
12. Antidiuretic hormone (ADH) is increased by which of the
following?
A) A hyperosmotic extracellular fluid in the hypothalamus
B) A hyperosmotic extracellular fluid in the adenohypophysis
C) A hypoosmotic extracellular fluid in the hypothalamus
D) A hypoosmotic extracellular fluid in the adenohypophysis
E) A hypoosmotic fluid in the atria of the heart
13. In an individual with panhypopituitarism, which selection below
best describes the plasma hormone changes that would occur?
A) ↓GHRH, ↓somatostatin, ↓growth hormone, ↓somatomedin C
B) ↓GHRH, ↓somatostatin, ↓growth hormone, ↑somatomedin C
C) ↑GHRH, ↑somatostatin, ↑growth hormone, ↓somatomedin C
D) ↑GHRH, ↑somatostatin, ↓ growth hormone, ↓somatomedin C
E) ↑GHRH, ↓somatostatin, ↓growth hormone, ↓somatomedin C
14. Which of the following could inhibit the initiation of labor?
A) Administration of an antagonist of the actions of
progesterone
B) Administration of LH
C) Administration of an antagonist of PGE2 effects
D) Mechanically dilating and stimulating the cervix
E) Administration of oxytocin
15. A patient has nephrogenic diabetes insipidus. Which of the
following would either be expected or a suggested intervention?
A) Decreased plasma sodium concentration
B) Increased secretion of ADH from the supraoptic nuclei
C) High urine osmolality
D) Increased AVPR2 function
E) Decrease secretion of ADH from the supraoptic and
paraventricular nuclei
16. Which of the following would most likely cause a decrease in the
release of thyroid-stimulating hormone?
A) Decreased iodinase enzyme
B) Decreased iodine pump activity in thyroid gland
C) Decreased body temperature
D) Increased thyrotropin releasing hormone
E) Increased plasma thyroxine by venous infusion
17. The increased cardiac output caused by elevated circulating levels
of thyroid hormones is most likely caused by
A) Direct actions of thyroid-stimulating hormone on the heart
y g
B) Direct actions of thyroid-stimulating hormone on the brain
C) An increase in the metabolic demand of the tissues
D) An increase in plasma cholesterol and triglycerides
E) An increase in total body weight
18. If a radioimmunoassay is properly conducted and the amount of
radioactive hormone bound to antibody is low, what would this
result indicate?
A) Plasma levels of endogenous hormone are high
B) Plasma levels of endogenous hormone are low
C) More antibody is needed
D) Less radioactive hormone is needed
19. Which of the following depicts the most likely sequence of events
in an individual exposed to cold?
A) ↑Thyrotropin-releasing hormone, ↑thyroid-stimulating
hormone, ↑thyroxine
B) ↑Thyrotropin-releasing hormone, ↓thyroid-stimulating
hormone, ↑thyroxine
C) ↑Thyroid-stimulating hormone, ↑thyrotropin-releasing
hormone, ↑thyroxine
D) ↑Thyroid-stimulating hormone, ↓thyrotropin-releasing
hormone, ↑thyroxine
E) ↑Thyroxine, ↑thyrotropin-releasing hormone, ↑thyroidstimulating hormone
20. Spermatogenesis is regulated by a negative feedback control
system in which FSH stimulates the steps in sperm cell formation.
Which negative feedback signal associated with sperm cell
production inhibits pituitary formation of FSH?
A) Testosterone
B) Inhibin
C) Estrogen
D) LH
21. In an individual with a thyroid hormone producing adenoma, one
might expect which of the following?
A) ↑ T4, ↓ T3, ↓ TRH, ↓ TSH
B) ↑ T4, ↑ T3, ↓ TRH, ↓ TSH
C) ↑ T4, ↑ T3, ↑ TRH, ↓ TSH
D) ↑ T4, ↑ T3, ↓ TRH, ↑ TSH
E) ↓ T4, ↑ T3, ↓ TRH, ↓ TSH
22. When do progesterone levels rise to their highest point during the
female hormonal cycle?
A) Between ovulation and the beginning of menstruation
B) Immediately before ovulation
C) When the blood concentration of LH is at its highest point
D) When 12 primary follicles are developing to the antral stage
23. You suspect thyroid disease in a female patient. Based on the
plasma values below, which of the following would be expected?
[TSH] Total [T4] [TBG]
Normal range
0.4-5.5 mU/l 5.6-14.7 µg/dl 1.7-3.6 µg/dl
Patient data
9.3 2.3 3.0
A) Graves’ disease
B) Secondary hyperthyroidism
C) Hashimoto’s disease
D) Secondary hypothyroidism
E) Euthyroid pregnant
24. Which of the following enzymes catalyzes the conversion of
cholesterol to pregnenolone?
A) Aldosterone synthase
B) Lipoprotein lipase
C) Hormone sensitive lipase
D) 11β-Hydroxylase
E) Cholesterol desmolase
25. Which of the following would most likely occur if plasma
aldosterone levels were low?
A) Hyperkalemia
B) Hypokalemia
C) Hypernatremia
D) Hypertension
26. A professional athlete in her mid-20s has not had a menstrual
cycle for 5 years, although a bone density scan revealed normal
skeletal mineralization. Which fact may explain these
observations?
A) She consumes a high-carbohydrate diet
B) Her grandmother sustained a hip fracture at age 79 years
C) Her blood pressure is higher than normal
D) Her plasma estrogen concentration is very low
E) She has been taking anabolic steroid supplements for 5 years
27. During a chronic infusion of aldosterone in an experimental
animal model, one would expect which of the following?
A) ↑Blood pressure, ↔extracellular fluid volume, ↓urinary
sodium excretion
B) ↑Blood pressure, ↓extracellular fluid volume, ↔urinary
sodium excretion
C) ↑Blood pressure, ↔extracellular fluid volume, ↑urinary
sodium excretion
D) ↑Blood pressure, ↑extracellular fluid volume, ↔urinary
sodium excretion
E) ↑Blood pressure, ↔extracellular fluid volume, ↔urinary
sodium excretion
28. In the circulatory system of a fetus, which of the following is
greater before birth than after birth?
A) Arterial Po2
B) Right atrial pressure
C) Aortic pressure
D) Left ventricular pressure
29. In response to a physiological stimulus such as the stress of taking
an important quiz, which of the following reflects the most likely
sequence of events?
A) ↑Cortisol, ↑corticotropin, ↑corticotropin-releasing hormone
B) ↑Corticotropin-releasing hormone, ↑corticotropin, ↑cortisol
C) ↑Cortisol, ↓corticotropin, ↑corticotropin-releasing hormone
D) ↑Corticotropin-Releasing hormone, ↑corticotropin, ↓cortisol
E) ↑Cortisol, ↑corticotropin, ↓corticotropin-releasing hormone
30. Which of the following best characterizes the metabolic actions of
cortisol?
A) ↑Muscle glucose uptake, ↑muscle amino acid uptake,
↑adipose tissue fat uptake
B) ↑Muscle glucose uptake, ↓muscle amino acid uptake, ↑adipose
tissue fat uptake
C) ↓Muscle glucose uptake, ↓muscle amino acid uptake,
↑adipose tissue fat uptake
D) ↓Muscle glucose uptake, ↑muscle amino acid uptake,
↓adipose tissue fat uptake
p p
E) ↓Muscle glucose uptake, ↓muscle amino acid uptake, ↓adipose
tissue fat uptake
31. Which of the following is most likely to occur as a result of chronic
hyperglycemia associated with untreated type 1 diabetes mellitus?
A) Increased intracellular fluid volume
B) Decreased urinary glucose
C) Metabolic alkalosis
D) Osmotic diuresis and polyuria
E) Improved eyesight
32. Which enzyme in the cytochrome P450 steroid synthesis cascade is
directly responsible for estradiol synthesis?
A) 17-Beta-hydroxysteroid dehydrogenase
B) 5-Alpha reductase
C) Aromatase
D) Side chain cleavage enzyme
33. Which of the following is greater after birth than before birth?
A) Flow through the foramen ovale
B) Pressure in the right atrium
C) Flow through the ductus arteriosus
D) Aortic pressure
34. Immediately after consuming a meal consisting of a large burger,
French fries, onion rings, and a diet cola, one might expect a
DECREASE in which of following?
A) Amino acid transport into cells
B) Fay acid synthesis
C) Hormone sensitive lipase
D) Liver glycogen
E) Cell permeability to glucose
35. In an individual with untreated insulin dependent diabetes
mellitus (type 1), one would expect which of the following?
A) ↑Plasma free fay acids, ↓liver glycogen, ↑skeletal muscle
mass
B) ↑Plasma free fay acids, ↓liver glycogen, ↓skeletal muscle
mass
C) ↑Plasma free fay acids, ↑liver glycogen, ↓skeletal muscle
mass
D) ↓Plasma free fay acids, ↓liver glycogen, ↑skeletal muscle
mass
E) ↓Plasma free fay acids, ↑liver glycogen, ↓skeletal muscle
mass
36. Which of the following changes would be expected to help
maintain plasma glucose in the postabsorptive?
A) ↓Insulin, ↑glucagon, ↓growth hormone, ↓cortisol
B) ↓Insulin, ↑glucagon, ↑growth hormone, ↓cortisol
C) ↓Insulin, ↑glucagon, ↑growth hormone, ↑cortisol
D) ↑Insulin, ↓glucagon, ↓growth hormone, ↑cortisol
E) ↑Insulin, ↓glucagon, ↑growth hormone, ↑cortisol
37. For male differentiation to occur during embryonic development,
testosterone must be secreted from the testes. What stimulates the
secretion of testosterone during embryonic development?
A) LH from the maternal pituitary gland
B) hCG
C) Inhibin from the corpus luteum
D) GnRH from the embryo’s hypothalamus
38. Which of the following best describes insulin?
A) Lipid-soluble hormone tightly bound to plasma proteins
B) Peptide hormone that activates an intracellular receptor
C) Peptide hormone that activates a G-coupled protein receptor
D) Peptide hormone that activates an enzyme-linked receptor
E) Steroid hormone that activates an enzyme-linked receptor
39. If one were to experience a sudden decrease in extracellular fluid
calcium, which of the following would most likely be the first
physiological response to buffer the change in calcium?
A) Increased calcium absorption in the gut
B) Decreased phosphate absorption in the gut
C) Increased parathyroid hormone from the anterior pituitary
D) Decreased renal excretion of phosphate
E) Increased exchange of calcium with the bone fluid
40. As menstruation ends, estrogen levels in the blood rise rapidly.
What is the source of the estrogen?
A) Corpus luteum
B) Developing follicles
C) Endometrium
D) Stromal cells of the ovaries
E) Anterior pituitary gland
41. A 30-year-old woman reports to the clinic for a routine physical
examination. The examination reveals she is pregnant. Her plasma
p g p
levels of TSH are high, but her total thyroid hormone concentration
is normal. Which of the following best reflects the patient’s clinical
state?
A) Graves’ disease
B) Hashimoto’s disease
C) A pituitary tumor secreting TSH
D) A hypothalamic tumor secreting thyrotropin-releasing
hormone (TRH)
E) The patient is taking thyroid extract
42. Which of the following would be expected in a patient with
chronic renal failure?
Plasma[1,25-(OH)2D] Plasma [PTH] Bone Resorption
A) ↑ ↑ ↑
B) ↑ ↑ ↓
C) ↑ ↓ ↓
D) ↓ ↓ ↑
E) ↓ ↑ ↓
F) ↓ ↑ ↑
43. A female athlete who took testosterone-like steroids for several
months stopped having normal menstrual cycles. What is the best
explanation for this observation?
A) Testosterone stimulates inhibin production from the corpus
luteum
B) Testosterone binds to receptors in the endometrium, resulting
in the failure of the endometrium to develop during the
normal cycle
C) Testosterone binds to receptors in the anterior pituitary that
stimulate the secretion of FSH and LH
D) Testosterone inhibits the hypothalamic secretion of GnRH
and the pituitary secretion of LH and FSH
44. An experiment is conducted in which ADH is administered at
hour 3 to four subjects (A to D). In the above figure, which lines
most likely reflect the response to ADH administration in a normal
patient and in a patient with central diabetes insipidus?
Normal Central Diabetes Insipidus
A) B A
B) B D
C) D A
D) D B
45. Which of the following decreases the resistance in the arteries
leading to the sinuses of the penis?
A) Stimulation of the sympathetic nerves innervating the arteries
B) NO
C) Inhibition of activity of the parasympathetic nerves leading to
the arteries
D) All the above
46. Using the three following statements, select the best answer.
1. Hydroxyapatite is the major crystalline salt in calcified bone
2. An osteon is made up of concentric layers of bone called
lamellae
3. Osteocytes are the major cells responsible for the formation of
new bone tissue
A) Only statement 1 is correct
B) Statement 1 and 2 are correct
C) Statements 1 and 3 are correct
D) All statements are correct
E) No statements are correct
47. All of the following statements about parathyroid hormone are
true EXCEPT one. Which one is the EXCEPTION?
A) PTH directly activates osteoblasts and osteocytes
B) PTH inhibits the production of vitamin D hormones
C) PTH promotes bone resorption in response to decreased
plasma calcium
D) PTH promotes the movement of calcium from bone fluid to
the extracellular fluid
E) PTH promotes calcium reabsorption in the renal distal tubule
and collecting duct
48. A 46-year-old man has “puffy” skin and is lethargic. His plasma
TSH concentration is low and increases markedly when he is given
TRH. What is the most likely diagnosis?
A) Hyperthyroidism due to a thyroid tumor
B) Hyperthyroidism due to an abnormality in the hypothalamus
C) Hypothyroidism due to an abnormality in the thyroid
D) Hypothyroidism due to an abnormality in the hypothalamus
E) Hypothyroidism due to an abnormality in the pituitary
49. Negative feedback on FSH release from the anterior pituitary in
men that results in a reduction in estradiol production is due to
which hormone?
A) Progesterone
B) Estradiol
C) Testosterone
D) Inhibin
50. During the first few years after menopause, FSH levels are
normally extremely high. A 56-year-old woman completed
menopause 3 years ago. However, she is found to have low levels
of FSH in her blood. What is the best explanation for this finding?
A) She has been receiving hormone replacement therapy with
estrogen and progesterone since she completed menopause
B) Her adrenal glands continue to produce estrogen
C) Her ovaries continue to secrete estrogen
D) She took birth control pills for 20 years before menopause
51. Blockade of what receptors will prolong erection in a man?
A) Estrogen receptors
B) Cholesterol receptors
C) Muscarinic receptors
D) Phosphodiesterase-5 receptors
52. Which of the following pairs of hormones and the corresponding
action is incorrect?
A) Glucagon—increased glycogenolysis in liver
B) Glucagon—increased glycogenolysis in skeletal muscle
C) Glucagon—increased gluconeogenesis
D) Cortisol—increased gluconeogenesis
E) Cortisol—decreased glucose uptake in muscle
53. A large dose of insulin is administered intravenously to a patient.
Which set of hormonal changes is most likely to occur in the
plasma in response to the insulin injection?
Growth Hormone Glucagon Epinephrine
A) ↑ ↓ ↔
B) ↔ ↑ ↑
C) ↑ ↑ ↑
D) ↓ ↑ ↑
E) ↓ ↓ ↔
54. What is a frequent cause of delayed breathing at birth?
A) Fetal hypoxia during the birth process
B) Maternal hypoxia during the birth process
C) Fetal hypercapnia
D) Maternal hypercapnia
55. Which hormone is largely unbound to plasma proteins?
A) Cortisol
B) T4
C) ADH
D) Estradiol
E) Progesterone
56. What is the mechanism by which the zona pellucida becomes
“hardened” after penetration of a sperm cell to prevent a second
sperm from penetrating?
A) A reduction in estradiol
B) The proteins released from the acrosome of the sperm
C) An increase in intracellular calcium in the oocyte
D) An increase in testosterone that affects the sperm
57. Why is milk produced by a woman only after delivery, not before?
A) Levels of LH and FSH are too low during pregnancy to
support milk production
B) High levels of progesterone and estrogen during pregnancy
suppress milk production
C) The alveolar cells of the breast do not reach maturity until
after delivery
D) High levels of oxytocin are required for milk production to
begin, and oxytocin is not secreted until the baby stimulates
the nipple
58. Which of the following increases the rate of excretion of calcium
ions by the kidney?
A) A decrease in calcitonin concentration in the plasma
B) An increase in phosphate ion concentration in the plasma
C) A decrease in the plasma level of PTH
D) Metabolic alkalosis
59. A patient has hyperthyroidism due to a pituitary tumor. Which set
of physiological changes would be expected?
Thyroglobulin Synthesis Heart Rate Exophthalmos
A) ↑ ↑ +
B) ↑ ↑ −
C) ↑ ↓ +
D) ↓ ↓ +
E) ↓ ↓ −
F) ↓ ↑ −
60. A 25-year-old man is severely injured when hit by a speeding
vehicle and loses 20% of his blood volume. Which set of
physiological changes would be expected to occur in response to
the hemorrhage?
Atrial Stretch Receptor Activity Arterial Baroreceptor Activity ADH Secretion
A) ↓ ↓ ↑
B) ↓ ↓ ↓
C) ↔ ↑ ↑
D) ↑ ↑ ↑
E) ↑ ↑ ↓
61. If a woman has a tumor that is secreting large amounts of estrogen
from the adrenal gland, which of the following will occur?
A) Progesterone levels in the blood will be very low
B) Her LH secretion rate will be totally suppressed
C) She will not have normal menstrual cycles
D) Her bones will be normally calcified
E) All the above
62. When compared with the postabsorptive state, which set of
metabolic changes would most likely occur during the
postprandial state?
Hepatic Glucose Uptake Muscle Glucose Uptake Hormone-Sensitive Lipase Activity
A) ↑ ↑ ↑
B) ↑ ↓ ↑
C) ↓ ↑ ↓
D) ↑ ↑ ↓
E) ↓ ↑ ↑
63. Very early in embryonic development, testosterone is formed
within male embryos. What is the function of this hormone at this
stage of development?
A) Stimulation of bone growth
B) Stimulation of development of male sex organs
C) Stimulation of development of skeletal muscle
D) Inhibition of LH secretion
64. During spermatogenesis, estrogen is produced by
A) Leydig cells in response to FSH
B) Sertoli cells in response to FSH
C) Leydig cells in response to LH
D) Sertoli cells in response to LH
65. A patient arrives in the emergency department apparently in
cardiogenic shock due to a massive heart aack. His initial arterial
blood sample reveals the following concentrations of ions and pH
level:
Sodium 137 mmol/l
Bicarbonate
14 mmol/l
Free calcium 2.8 mmol/l
Potassium 4.8 mmol/l
pH 7.16
To correct the acidosis, the aending physician begins an infusion of
sodium bicarbonate and after 1 hour obtains another blood sample, which
reveals the following values:
Sodium 138 mmol/l
Bicarbonate
22 mmol/l
Free calcium 2.3 mmol/l
Potassium 4.5 mmol/l
pH 7.34
What is the cause of the decrease in calcium ion concentration?
A) The increase in arterial pH resulting from the sodium
bicarbonate infusion inhibited PTH secretion
B) The increase in pH resulted in the stimulation of osteoblasts,
which removed calcium from the circulation
C) The increase in pH resulted in an elevation in the concentration
of HPO4 −, which shifted the equilibrium between HPO4 − and
Ca2+
toward CaHPO4
D) The increase in arterial pH stimulated the formation of 1,25-
dihydroxycholecalciferol, which resulted in an increased rate of
absorption of calcium from the gastrointestinal tract
66. The prostate fluid contributes the bulk of the volume of semen,
which includes
A) Calcium, citrate, phosphate, and profibrinolysin
B) Fructose, citric acid, prostaglandins, and fibrinogen
C) Sex hormones
D) Mucus
67. A 30-year-old woman is breastfeeding her infant. During suckling,
which hormonal response is expected in the woman?
A) Increased secretion of ADH from the supraoptic nuclei
B) Increased secretion of ADH from the paraventricular nuclei
C) Increased secretion of oxytocin from the paraventricular
nuclei
D) Decreased secretion of neurophysin
E) Increased plasma levels of both oxytocin and ADH
68. A 30-year-old man has Conn’s syndrome. Which set of
physiological changes is most likely to occur in this patient
compared with a healthy person?
Arterial Pressure Extracellular Fluid Volume Sodium Excretion
A) ↔ ↔ ↔
B) ↑ ↔ ↔
C) ↑ ↑ ↔
D) ↔ ↑ ↓
E) ↑ ↑ ↓
69. Which of the following is important in the process of capacitation
of sperm after ejaculation?
A) Microtubule reorganization
B) Increased testosterone secretion by spermatozoa
C) Washout of inhibitory factors
D) Influx of glucose
70. Dehydroepiandrosterone sulfate (DHEAS), the precursor for the
high levels of estradiol that occur in pregnancy, is made in what
tissue?
A) Fetal adrenal gland
B) Ovary of the mother
C) Placenta
D) Adrenal gland of the mother
71. What is the consequence of sporadic nursing of the neonate by the
mother?
A) An increase in prolactin-releasing hormone
B) An increase in oxytocin
C) Lack of birth control
D) Lack of prolactin surge
72. Which of the following would be associated with parallel changes
in aldosterone and cortisol secretion?
A) Addison’s disease
B) Cushing’s disease
C) Cushing’s syndrome (adrenal tumor)
D) A low-sodium diet
E) Administration of a converting enzyme inhibitor
73. The process of spermatogenesis begins with spermatogonia and
results in which of the following?
A) 1 diploid spermatid
B) 4 diploid spermatids
C) 1 haploid spermatid
D) 2 haploid spermatids
E) 4 haploid spermatids
74. RU486 causes abortion if it is administered before or soon after
implantation. What is the specific effect of RU486?
A) It binds to LH receptors, stimulating the secretion of
progesterone from the corpus luteum
B) It blocks progesterone receptors so that progesterone has no
effect within the body
C) It blocks the secretion of FSH by the pituitary
D) It blocks the effects of oxytocin receptors in the uterine
muscle
75. A 55-year-old man has developed the syndrome of inappropriate
antidiuretic hormone secretion due to carcinoma of the lung.
Which physiological response would be expected?
A) Increased plasma osmolality
B) Inappropriately low urine osmolality (relative to plasma
osmolality)
C) Increased thirst
D) Decreased secretion of ADH from the pituitary gland
p y g
76. During pregnancy, the uterine smooth muscle is quiescent. During
the ninth month of gestation, the uterine muscle becomes
progressively more excitable. What factor contributes to the
increase in excitability?
A) Placental estrogen synthesis rises to high rates
B) Progesterone synthesis by the placenta decreases
C) Uterine blood flow reaches its highest rate
D) PGE2 synthesis by the placenta decreases
E) Activity of the fetus falls to low levels
77. A 20-year-old woman is not having menstrual cycles. Her plasma
progesterone concentration is found to be minimal. What is the
explanation for the low level of progesterone?
A) LH secretion rate is elevated
B) LH secretion rate is suppressed
C) FSH secretion rate is suppressed
D) No corpus luteum is present
E) High inhibin concentration in the plasma has suppressed
progesterone synthesis
78. Before the preovulatory surge in LH, granulosa cells of the follicle
secrete which hormone?
A) Testosterone
B) Progesterone
C) Estrogen
D) Inhibin
Questions 79 and 80
79. Based on the above figure, which set of curves most likely reflects
the responses in a healthy individual and in patients with type 1 or
type 2 diabetes mellitus (Diabetes mellitus)?
Healthy Type 1 Diabetes mellitus Type 2 Diabetes mellitus
A) 3 2 1
B) 1 2 3
C) 1 3 2
D) 2 1 3
E) 2 3 1
80. Based on the above figure, which set of curves most likely reflects
the responses in a healthy person and in a patient in the early
stages of Cushing’s syndrome?
Healthy Cushing’s Syndrome
A) 3 2
B) 1 2
C) 1 3
D) 2 1
E) 2 3
81. Which hormone activates enzyme-linked receptors?
A) ADH
B) Insulin
C) ACTH
D) PTH
E) Aldosterone
82. Which of the following is produced by the trophoblast cells during
the first 3 weeks of pregnancy?
A) Estrogen
B) LH
C) Oxytocin
D) hCG
E) None of the above
83. Which of the following is higher in a neonate than in a fetus?
A) Flow through the foramen ovale
B) Right atrial pressure
C) Flow through the ductus arteriosus
D) Aortic pressure
84. Which finding is most likely in a patient who has myxedema?
A) Somnolence
B) Palpitations
C) Increased respiratory rate
D) Increased cardiac output
E) Weight loss
85. At birth, a large, well-nourished baby is found to have a plasma
glucose concentration of 17 mg/dl (normal is 80 to 100 mg/dl) and a
plasma insulin concentration twice the normal value. What is the
explanation for these findings?
A) The neonate experienced in utero malnutrition
B) The mother was malnourished during pregnancy
C) The mother has diabetes, with poorly controlled
hyperglycemia
D) The mother is obese
86. Degradation of the corpus luteum is prevented by which of the
following?
A) Increased estrogen secretion by the developing placenta
B) Release of hCG from the trophoblasts
C) Forward positive regulation by LH
D) Placental derived prolactin
87. Which of the following stimulates the secretion of PTH?
A) An increase in extracellular calcium ion activity above the
normal value
B) An increase in calcitonin concentration
C) Respiratory acidosis
D) Increased secretion of PTH-releasing hormone from the
hypothalamus
E) None of the above
88. A 40-year-old woman consumes a high-potassium diet for several
weeks. Which hormonal change is most likely to occur?
A) Increased secretion of DHEA
B) Increased secretion of cortisol
C) Increased secretion of aldosterone
D) Increased secretion of ACTH
E) Decreased secretion of CRH
89. After implantation into the uterus, nutrition of the blastocyst
comes from which structure?
A) Placenta
B) Decidua
C) Glomerulosa cells
D) Corpus luteum
90. Which hormone is not stored in its endocrine-producing gland?
A) T4
B) PTH
C) Aldosterone
D) ACTH
E) Insulin
91. A young woman comes to the emergency department with a
vertebral compression fracture. Radiographs of the spine indicate
generalized demineralization. She is vegetarian, does not smoke or
drink alcohol, and has a normal plasma potassium concentration of
5.4 mEq/l, a sodium concentration of 136 mEq/l, and a plasma
calcium concentration of 7.0 mg/dl. Her vitamin D3
value is several
times greater than normal, although her 1,25-
dihydroxycholecalciferol concentration is at the lower limit of
detectability. She has been in renal failure for the past 5 years and
undergoes hemodialysis three times each week. What is the cause
of her low 1,25-dihydroxycholecalciferol level?
A) Metabolic acidosis
B) Metabolic alkalosis
C) She is unable to form 1,25-dihydroxycholecalciferol because
of her extensive kidney disease
D) She is undergoing dialysis with a dialysis fluid that does not
contain calcium
E) She is taking receiving calcium supplements
92. A placenta is incapable of synthesizing which hormones?
A) Estrogen
B) Progesterone
C) Androgens
D) Estriol
93. Which of the following hormones is most closely associated with
the secretory phase of the endometrial cycle?
A) Progesterone
B) Estrogen
C) FSH
D) LH
E) Inhibin
94. Which finding would likely be reported in a patient with a
deficiency in iodine intake?
A) Weight loss
B) Nervousness
C) Increased sweating
D) Increased synthesis of thyroglobulin
E) Tachycardia
95. A 37-year-old woman presents to her physician with an enlarged
thyroid gland and high plasma levels of T4
and T3
. Which of the
following is likely to be decreased?
A) Heart rate
B) Cardiac output
C) Peripheral vascular resistance
D) Ventilation rate
E) Metabolic rate
96. Before intercourse, a woman irrigates her vagina with a solution
that lowers the pH of the vaginal fluid to 4.5. What will be the
effect on sperm cells in the vagina?
A) The metabolic rate will increase
B) The rate of movement will decrease
C) The formation of PGE2 will increase
D) The rate of oxygen consumption will increase
97. Which hormonal responses would be expected after a meal high in
protein?
Insulin Glucagon Growth Hormone
A) ↑ ↑ ↓
B) ↑ ↑ ↑
C) ↑ ↓ ↓
D) ↓ ↓ ↑
E) ↓ ↑ ↑
98. Men who take large doses of testosterone-like androgenic steroids
for long periods are sterile in the reproductive sense of the word.
What is the explanation for this finding?
A) High levels of androgens bind to testosterone receptors in the
Sertoli cells, resulting in overstimulation of inhibin formation
B) Overstimulation of sperm cell production results in the
formation of defective sperm cells
C) High levels of androgen compounds inhibit the secretion of
GnRH by the hypothalamus, resulting in the inhibition of LH
and FSH release by the anterior pituitary
D) High levels of androgen compounds produce hypertrophic
dysfunction of the prostate gland
99. Cortisone is administered to a 30-year-old woman for the
treatment of an autoimmune disease. Which of the following is
most likely to occur?
A) Increased ACTH secretion
B) Increased cortisol secretion
C) Increased insulin secretion
D) Increased muscle mass
E) Hypoglycemia between meals
100. In the hypothalamic-pituitary-gonadal axis of the female, what is
the follicular cell type that produces inhibin?
A) Cytotrophoblasts
B) Synthiotrophoblasts
C) Granulosa
D) Thecal
101. The function of which of the following is increased by an elevated
parathyroid hormone concentration?
A) Osteoclasts
B) Hepatic formation of 25-hydroxycholecalciferol
C) Phosphate reabsorptive pathways in the renal tubules
D) All the above
102. Which statement about peptide or protein hormones is usually
true?
A) They have longer half-lives than steroid hormones
B) They have receptors on the cell membrane
C) They have a slower onset of action than both steroid and
thyroid hormones
D) They are not stored in endocrine-producing glands
103. Which set of physiological changes would be most likely to occur
in a patient with acromegaly?
Pituitary Mass Kidney Mass Femur Length
A) ↓ ↓ ↑
B) ↓ ↑ ↑
C) ↑ ↔ ↔
D) ↑ ↑ ↔
E) ↑ ↑ ↑
104. Cortisol and GH are most dissimilar in their metabolic effects on
which of the following?
A) Protein synthesis in muscle
B) Glucose uptake in peripheral tissues
C) Plasma glucose concentration
D) Mobilization of triglycerides
105. Why do infants of mothers who had adequate nutrition during
pregnancy not require iron supplements or a diet rich in iron until
about 3 months of age?
A) Growth of the infant does not require iron until after the third
month
B) The fetal liver stores enough iron to meet the infant’s needs
until the third month
C) Synthesis of new red blood cells begins after 3 months
D) Muscle cells that develop before the third month do not
contain myoglobin
106. Cortisone is administered to a patient for the treatment of an
autoimmune disease. Which of the following would least likely
occur in response to the cortisone treatment?
A) Hypertrophy of the adrenal glands
B) Increased plasma levels of C-peptide
C) Decreased CRH secretion
D) Increased blood pressure
E) Hyperglycemia
107. All of the following accurately describe the regulation of the
female sexual cycle EXCEPT one. Which one is the EXCEPTION?
A) Estradiol inhibits GnRH release during the post ovulatory
phase
B) Progesterone increases GnRH release during the post
ovulatory phase
C) Estradiol increases LH in the days immediately preceding
ovulation
D) Falling progesterone and estrogen late in the luteal phase
allows LH and FSH to rise
E) LH and FSH increase estradiol release during the follicular
phase
108. If a male is born without a penis and testes, a defect is likely in
which gene on the Y chromosome?
A) ERE—estrogen response element
B) ARE—androgen response element
C) SRY—affecting Sertoli cells
D) ERG—early response genes
y p g
109. Where does fertilization normally take place?
A) Uterus
B) Cervix
C) Ovary
D) Ampulla of the fallopian tubes
110. Which finding is most likely to occur in a patient who has
uncontrolled type 1 diabetes mellitus?
A) Decreased plasma osmolality
B) Increased plasma volume
C) Increased plasma pH
D) Increased release of glucose from the liver
E) Decreased rate of lipolysis
111. GH secretion would most likely be suppressed under which
condition?
A) Acromegaly
B) Gigantism
C) Deep sleep
D) Exercise
E) Acute hyperglycemia
112. Pregnenolone is not in the biosynthetic pathway of which
substance?
A) Cortisol
B) Estrogen
C) Aldosterone
D) 1,25(OH)2D
E) DHEA
113. Two days before the onset of menstruation, secretions of FSH and
LH reach their lowest levels. What is the cause of this low level of
secretion?
A) The anterior pituitary gland becomes unresponsive to the
stimulatory effect of GnRH
B) Estrogen from the developing follicles exerts a feedback
inhibition on the hypothalamus
C) The rise in body temperature inhibits hypothalamic release of
GnRH
D) Secretion of estrogen, progesterone, and inhibin by the corpus
luteum suppresses hypothalamic secretion of GnRH and
pituitary secretion of FSH
114. Which condition contributes to “sodium escape” in persons with
Conn’s syndrome?
A) Decreased plasma levels of atrial natriuretic peptide
B) Increased plasma levels of angiotensin II
C) Decreased sodium reabsorption in the collecting tubules
D) Increased arterial pressure
115. Which of the following most accurately describes events in the
female sexual cycle?
A) FSH causes the development of the corpus luteum
B) Estrogen and LH have a positive feedback relationship during
the late follicular phase
C) Estrogens are primarily produced by theca cells in the
developing ovary
D) During the luteal phase, estrogen increases to a greater degree
than progesterone
E) LH is most responsible for the development of primary
follicles
116. A 30-year-old woman reports to the clinic for a routine physical
examination, which reveals she is pregnant. Her plasma levels of
TSH are high, but her total T4
concentration (protein bound and
free) is normal. Which of the following best reflects this patient’s
clinical state?
A) Graves’ disease
B) Hashimoto’s disease
C) A pituitary tumor that is secreting TSH
D) A hypothalamic tumor that is secreting TRH
E) The patient is taking thyroid extract
117. A man has a disease that destroyed only the motor neurons of the
spinal cord below the thoracic region. Which aspect of sexual
function would not be possible?
A) Arousal
B) Erection
C) Lubrication
D) Ejaculation
118. Which of the following is responsible for invasion of the uterus
and formation of the placenta?
A) Trophoblasts
B) Oocyte
C) Decidua
D) Endometrium
119. A sustained program of lifting heavy weights will increase bone
mass. What is the mechanism of this effect of weightlifting?
A) Elevated metabolic activity stimulates parathyroid hormone
secretion
B) Mechanical stress on the bones increases the activity of
osteoblasts
C) Elevated metabolic activity results in an increase in dietary
calcium intake
D) Elevated metabolic activity results in stimulation of calcitonin
secretion
120. The hormone most responsible for maintaining milk production
after parturition is
A) Estrogen
B) Progesterone
C) Oxytocin
D) Prolactin
E) Inhibin
121. Which of the following would be expected in a patient with a
genetic deficiency of 11-β-hydroxysteroid dehydrogenase type II?
A) Hyperkalemia
B) Hypertension
C) Increased plasma renin activity
D) Increased plasma [aldosterone]
E) Hyperglycemia
122. Which physiological response is greater for T3
than for T4?
A) Secretion rate from the thyroid
B) Plasma concentration
C) Plasma half-life
D) Affinity for nuclear receptors in target tissues
E) Latent period for the onset of action in target tissues
123. A “birth control” compound for men has been sought for several
decades. Which substance would provide effective sterility?
A) A substance that mimics the actions of LH
B) A substance that blocks the actions of inhibin
C) A substance that blocks the actions of FSH
D) A substance that mimics the actions of GnRH
124. For milk to flow from the nipple of the mother into the mouth of
the nursing infant, what must occur?
A) Myoepithelial cells must relax
B) Prolactin levels must fall
C) Oxytocin secretion from the posterior pituitary must take
place
D) The baby’s mouth must develop a strong negative pressure
over the nipple
E) All the above
125. A number of normal physiological changes occur during
pregnancy. Which of the following best describes one of these
changes in the mother?
A) Increase total peripheral resistance
B) Increased cardiac output
C) Decreased metabolic rate
D) Decreased body weight
E) Decreased uterine size
126. Which set of physiological changes would be expected in a
nondiabetic patient with Cushing’s disease?
Plasma Aldosterone Plasma Cortisol Plasma Insulin
A) ↑ ↑ ↑
B) ↑ ↑ ↔
C) ↑ ↔ ↔
D) ↔ ↔ ↑
E) ↔ ↑ ↔
F) ↔ ↑ ↑
127. When compared with the late-evening values typically observed
in normal subjects, plasma levels of both ACTH and cortisol would
be expected to be higher in which persons?
A) Normal subjects after waking in the morning
B) Normal subjects who have taken dexamethasone
C) Patients with Cushing’s syndrome (adrenal adenoma)
D) Patients with Addison’s disease
E) Patients with Conn’s syndrome
128. Which of the following conditions or hormones would most
likely increase GH secretion?
A) Hyperglycemia
B) Exercise
C) Somatomedin
D) Somatostatin
E) Aging
129. Which set of findings would be expected in a person maintained
on a long-term low-sodium diet?
Plasma [Aldosterone] Plasma [Atrial Natriuretic Peptide] Plasma [Cortisol]
A) ↑ ↑ ↔
B) ↑ ↓ ↓
C) ↑ ↓ ↔
D) ↔ ↔ ↔
E) ↓ ↓ ↓
F) ↓ ↑ ↓
130. What would be associated with parallel changes in aldosterone
and cortisol secretion?
A) Addison’s disease
B) Cushing’s disease
C) Cushing’s syndrome (ectopic ACTH-producing tumor)
D) A high-sodium diet
E) Administration of a converting enzyme inhibitor
131. Which blood vessel in the fetus has the highest PO 2?
A) Ductus arteriosus
B) Ductus venosus
C) Ascending aorta
D) Left atrium
132. A 59-year-old woman has osteoporosis, hypertension, hirsutism,
and hyperpigmentation. Magnetic resonance imaging indicates
that the pituitary gland is not enlarged. Which condition is most
consistent with these findings?
A) Pituitary ACTH-secreting tumor
B) Ectopic ACTH-secreting tumor
C) Inappropriately high secretion rate of CRH
D) Adrenal adenoma
E) Addison’s disease
133. Which set of findings is an inappropriate hypophysial hormone
response to the hypothalamic hormone listed?
Hypothalamic Hormone Secretion Hypophysial Hormone
A) Somatostatin ↓ GH
B) Dopamine ↑ Prolactin
C) GnRH ↑ LH
D) TRH ↑ TSH
E) CRH ↑ ACTH
134. A patient is administered sufficient T4
to increase plasma levels of
the hormone several fold. Which set of changes is most likely in
this patient after several weeks of T4
administration?
Respiratory Rate Heart Rate Plasma Cholesterol Concentration
A) ↑ ↑ ↑
B) ↑ ↑ ↓
C) ↑ ↓ ↑
D) ↓ ↓ ↑
E) ↓ ↑ ↓
135. Which of the following hormones is most critical for sustaining a
successful pregnancy, even up through week 12 of gestation?
A) Estrogen
B) Progesterone
C) hCG
D) GnRH
E) Inhibin
136. What causes menopause?
A) Reduced levels of gonadotropic hormones secreted from the
anterior pituitary gland
B) Reduced responsiveness of the follicles to the stimulatory
effects of gonadotropic hormones
C) Reduced rate of secretion of progesterone from the corpus
luteum
D) Reduced numbers of follicles available in the ovary for
stimulation by gonadotropic hormones
137. What does not increase when insulin binds to its receptor?
A) Fat synthesis in adipose tissue
B) Protein synthesis in muscle
C) Glycogen synthesis
D) Gluconeogenesis in the liver
E) Intracellular tyrosine kinase activity
138. Release of which hormone is an example of neuroendocrine
secretion?
F) GH
G) Cortisol
H) Oxytocin
I) Prolactin
J) ACTH
139. The ability of a fetus to effectively use the relatively low maternal
Po2 is facilitated by which of the following?
A) Decreased glucose transport in the placental villi
B) Increased production of amniotic fluid
C) Increased total fetal hemoglobin concentration
D) Decreased placental membrane permeability
E) Decreased fetal hemoglobin binding capacity
140. Inhibition of the iodide pump would be expected to cause which
change?
A) Increased synthesis of T4
B) Increased synthesis of thyroglobulin
C) Increased metabolic rate
D) Decreased TSH secretion
E) Extreme nervousness
141. Before implantation, the blastocyst obtains its nutrition from
uterine endometrial secretions. How does the blastocyst obtain
nutrition during the first week after implantation?
A) It continues to derive nutrition from endometrial secretions
B) The cells of the blastocyst contain stored nutrients that are
metabolized for nutritional support
C) The placenta provides nutrition derived from maternal blood
D) The trophoblast cells digest the nutrient-rich endometrial cells
and then absorb their contents for use by the blastocyst
142. Which pituitary hormone has a chemical structure most similar
to that of ADH?
A) Oxytocin
B) ACTH
C) TSH
D) FSH
E) Prolactin
143. Which option would not be efficacious in the treatment of
patients with type 2 diabetes?
A) Glucocorticoids
B) Insulin injections
C) Thiazolidinediones
D) Sulfonylureas
E) Weight loss
144. Which of the following is most likely to occur in the early stages
of type 2 diabetes?
A) Increased insulin sensitivity
B) Decreased hepatic glucose output
C) Increased plasma levels of C-peptide
D) Increased plasma [β-hydroxybutyric acid]
E) Hypovolemia
145. What is the most common cause of respiratory distress syndrome
in neonates born at 7 months’ gestation?
A) Pulmonary edema due to pulmonary arterial hypertension
B) Formation of a hyaline membrane over the alveolar surface
C) Failure of the alveolar lining to form adequate amounts of
surfactant
D) Excessive permeability of the alveolar membrane to water
146. Which of the following is an expected circulatory change that
occurs after birth?
A) Opening of the ductus venosus
B) Opening of the foramen ovale
C) Opening of the ductus arteriosus
D) Closing of the ductus arteriosus
E) Closing of the inferior vena cava
147. A 45-year-old woman has a mass in the sella turcica that
compresses the portal vessels, disrupting pituitary access to
p p p g p y
hypothalamic secretions. The secretion rate of which hormone
would most likely increase in this patient?
A) ACTH
B) GH
C) Prolactin
D) LH
E) TSH
148. Which of the following is not produced by osteoblasts?
A) Alkaline phosphatase
B) RANK ligand
C) Collagen
D) Pyrophosphate
E) Osteoprotegerin
149. Which set of findings would be expected in a patient with
primary hyperparathyroidism?
Plasma [1,25-(OH)2D3] Plasma [Phosphate] Urinary Ca
2+ Excretion
A) ↑ ↑ ↑
B) ↑ ↓ ↑
C) ↑ ↓ ↓
D) ↓ ↓ ↑
E) ↓ ↑ ↓
F) ↓ ↑ ↑
150. A man who has been exposed to high levels of gamma radiation
is sterile due to destruction of the germinal epithelium of the
seminiferous tubules, although he has normal levels of
testosterone. Which of the following would be found in this
patient?
A) A normal secretory paern of GnRH
B) Normal levels of inhibin
C) Suppressed levels of FSH
D) Absence of Leydig cells
Questions 151 and 152
An experiment was conducted in which rats were injected with one of
two hormones or saline solution (control) for 2 weeks. Autopsies were then
performed, and organ weights were measured (in milligrams). Use this
information to answer Questions 151 and 152.
Control Hormone 1 Hormone 2
Pituitary
12.9 8.0 14.5
Thyroid 250 500 245
Adrenal glands 40 37 85
Body weight 300 152 175
151. What is hormone 1?
A) TRH
B) TSH
C) T4
D) ACTH
E) Cortisol
152. What is hormone 2?
A) TSH
B) T4
C) CRH
D) ACTH
E) Cortisol
153. An infant is born with yellow pigmentation in the skin and eyes.
What is the most likely cause of this?
A) Elevated circulating ACTH
B) Poor renal clearance of bilirubin
C) The lack of a fully functional liver at birth
D) Failure of the foramen ovale to close
E) Delayed onset of breathing
Answers
1. A) The freely circulating (unbound) hormone is the biologically active
hormone. Cholecalciferol is a prohormone and thus is not the
biologically active vitamin D hormone. In this question, cortisol, T4, and
aldosterone are all bound to carrier proteins.
TMP14 p. 929
2. D) Parasympathetic postganglionic fibers release acetylcholine that
activates muscarinic receptors on endothelium to produce NO and
increases cyclic guanosine monophosphate, which activates protein
kinase G, causing a reduction in intracellular calcium (also increasing
NO by positive feedback) and causing vasodilation.
TMP14 p. 1027
3. A) Estrogen compounds are believed to have an osteoblast-stimulating
effect. When the amount of estrogen in the blood falls to very low levels
after menopause, the balance between the bone-building activity of the
osteoblasts and the bone-degrading activity of the osteoclasts is tipped
toward bone degradation. When estrogen compounds are added as part
of hormone replacement therapy, the bone-building activity of the
osteoblasts is increased to balance the osteoclastic activity.
TMP14 pp. 949, 1045
4. A) ADH is made in the supraoptic nuclei of the hypothalamus. It is
transported in nerve fibers along with neurophysin carrier proteins that
pass through the pituitary stalk and terminate in the posterior pituitary.
TMP14 pp. 948–949
5. C) GH promotes several metabolic changes. These include a net increase
in amino acid uptake in the muscle and liver, a decrease in glucose
utilization and storage, and an increase in lipolysis. The net effect of GH
is to decrease glucose and lipid storage in adipose cells.
TMP14 pp. 943–944
6. A) Norepinephrine is released from the nerve terminals, and endothelin
is released from endothelial cells in the vasculature, causing
vasoconstriction of the vasculature.
TMP14 p. 1027
7. C) Type 2 diabetes mellitus is characterized by diminished sensitivity of
target tissues to the metabolic effects of insulin—that is, there is insulin
resistance. As a result, hepatic uptake of glucose is impaired, and
glucose release is enhanced. In muscle, the uptake of glucose is
impaired.
TMP14 pp. 985–986, 995
8. C) In acromegaly, high plasma levels of GH cause insulin resistance.
Consequently, glucose production by the liver is increased, and glucose
uptake by peripheral tissues is impaired.
TMP14 pp. 943–944, 996–997
9. A) During exercise, glucose utilization by muscle is increased, which is
largely independent of insulin.
TMP14 p. 985
10. A) Thecal cells do not have the capacity to produce estradiol because
they lack aromatase.
TMP14 pp. 1040, 1043, 1044
11. B) A very high concentration of testosterone in a female embryo will
induce formation of male genitalia. An adrenal tumor in the mother that
synthesizes testosterone at a high, uncontrolled rate could produce the
masculinizing effect.
TMP14 pp. 1043, 1044
12. A) Osmoreceptors in, or near, the hypothalamus are important
regulators of ADH. Hyperosmotic extracellular fluid causes the cells of
the hypothalamus to shrink and stimulates the release of ADH, which
promotes renal H2O reabsorption to restore the extracellular fluid to
isosmotic.
TMP14 p. 949
13. E) An individual with panhypopituitarism has generalized dysfunction
of the pituitary gland. GHRH from the hypothalamus is increased in an
aempt to restore the pituitary function. For similar reasons,
somatostatin is decreased. Because pituitary function is impaired,
growth hormone production is reduced, and because growth hormone
stimulates the production of somatomedin, its production is also
reduced.
TMP14 pp. 946–947
14. C) Antagonism of progesterone’s effects, dilation of the cervix, and
oxytocin all increase uterine smooth muscle excitability and facilitate
contractions and the onset of labor. LH would have no effect.
Prostaglandin E2
strongly stimulates uterine smooth muscle contraction
and is formed at an increasing rate by the placenta late in gestation.
TMP14 pp. 1064, 1066
15. B) AVPR2 function is impaired in patients with nephrogenic diabetes
insipidus, rendering ADH ineffective at increasing H2O reabsorption in
the distal nephron. This causes a compensatory increase in the release of
ADH from the supraoptic nuclei of the hypothalamus. Patients with
diabetes insipidus run the risk of developing hypernatremia, and they
produce a large volume of dilute urine.
TMP14 pp. 381, 439
16. E) Thyroxine (T4) is the major thyroid hormone, along with
triiodothyronine (T3). An increase in the thyroid hormones aenuates
the production of thyroid-stimulating hormone (TSH) through negative
feedback inhibition.
TMP14 pp. 958–959
17. C) Thyroid hormones cause a general increase in basal metabolic rate.
With an increased metabolic rate, there is an increased metabolic
demand of the tissues which is the primary determinant of cardiac
output.
TMP14 p. 957
18. A) In a radioimmunoassay, there is too lile antibody to completely
bind the radioactively tagged hormone and the hormone in the fluid
(plasma) to be assayed. Thus, there is competition between the labeled
and endogenous hormone for binding sites on the antibody.
Consequently, if the amount of radioactive hormone bound to antibody
is low, this finding would indicate that plasma levels of endogenous
hormone are high.
TMP14 p. 936
19. A) Cold exposure is an important physiological stimulus for the
production and release of the thyroid hormones. Cold causes the
hypothalamic production of thyrotropin-releasing hormone, which
stimulates thyrotropes of the anterior pituitary to release thyroidstimulating hormone (TSH). The increased TSH stimulates the
production of the thyroid hormones, including thyroxine, which helps
to relieve the physiological stress caused by the cold.
TMP14 pp. 958–959
20. B) The Sertoli cells of the seminiferous tubules secrete inhibin at a rate
proportional to the rate of production of sperm cells. Inhibin has a direct
inhibitory effect on anterior pituitary secretion of FSH. FSH binds to
specific receptors on the Sertoli cells, causing the cells to grow and
secrete substances that stimulate sperm cell production. The secretion of
inhibin thereby provides the negative feedback control signal from the
seminiferous tubules to the pituitary gland.
TMP14 p. 1033
21. B) A thyroid hormone–producing adenoma causes an increase in
thyroid hormones. Thus, one would expect an increase in both
circulating T4 and T3 caused by the adenoma. The increased T4 and T3
feeds back to inhibit the production and release of TRH from the
hypothalamus and TSH from the anterior pituitary to halt further
production of the thyroid hormones. However, the adenoma does not
respond to normal feedback regulation, and thus T3 and T4 remain
high.
TMP14 pp. 958–961
22. A) The corpus luteum is the only source of progesterone production,
except for minute quantities secreted from the follicle before ovulation.
The corpus luteum is functional between ovulation and the beginning of
menstruation, during which time the concentration of LH is suppressed
below the level achieved during the preovulatory LH surge.
TMP14 pp. 1046–1047
23. C) The high levels of TSH (outside the normal range) are indicative of
hypofunction of the thyroid, and this is further observed with the low
total T4. Thyroxine-binding globulin remains in the normal range,
making the best answer Hashimoto’s disease, which is the most
common form of hypothyroidism. Secondary hypothyroidism occurs in
response to failure of the pituitary gland to stimulate the thyroid.
Therefore, the high TSH rules out this possibility.
TMP14 pp. 961–962
24. E) Cholesterol desmolase is the key enzyme responsible for the
conversion of cholesterol to pregnenolone for the process of steroid
synthesis.
TMP14 p. 966
25. A) Aldosterone increases the Na+K+ ATPase in the basolateral
membrane of the principal cells and increases ENaC channels in the
luminal side. This creates a driving force for Na+ reabsorption and K+
excretion leading to hypokalemia. When aldosterone is low, K+
excretion is aenuated, leading to hyperkalemia.
TMP14 pp. 969–970
26. E) Anabolic steroids bind to testosterone receptors in the
hypothalamus, providing feedback inhibition of normal ovarian cycling
and preventing menstrual cycling as well as stimulation of osteoblastic
activity in the bones.
TMP14 pp. 1028, 1031
27. D) Chronically elevated aldosterone increases sodium and water
retention leading to an expansion of extracellular fluid volume.
Increased extracellular fluid leads to increased blood pressure, which
promotes pressure natriuresis, causing urinary sodium excretion to
come into balance. Thus, during a chronic infusion urinary sodium
excretion is not changed.
TMP14 p. 970
28. B) Right atrial pressure falls dramatically after the onset of breathing
because of a reduction in pulmonary vascular resistance, pulmonary
arterial pressure, and right ventricular pressure.
TMP14 pp. 1073–1075
29. B) Physiological stimuli for glucocorticoids, such as stress, cause the
hypothalamic production of corticotropin-releasing hormone (CRH).
CRH stimulates corticotropes from the anterior pituitary to release
corticotropin (or ACTH). Corticotropin promotes the production of
cortisol from the adrenal cortex to help alleviate the physiological
stressor.
TMP14 pp. 974–977
30. E) The metabolic actions of cortisol increase the availability of
circulating fuel sources in response to physiological stressors. Cortisol
impairs skeletal muscle glucose and amino acid uptake (although it
promotes hepatic amino acid uptake) and promotes lipolysis from
adipocytes. This has the net effect to increase plasma glucose, free fay
acids, and amino acids.
TMP14 pp. 972–973
31. D) Glucose is normally filtered in the glomerulus and reabsorbed in the
proximal tubule. However, during untreated type I diabetes, the amount
of filtered glucose exceeds (180 mg/dl) the reabsorptive capacity of the
proximal tubule, increasing urinary osmolarity. This causes an increase
in water filtration, leading to frequent urination (polyuria).
TMP14 pp. 995
32. C) Aromatase causes conversion of testosterone to estradiol.
TMP14 p. 1043
33. D) Because of the loss of blood flow through the placenta, systemic
vascular resistance doubles at birth, which increases the aortic pressure
as well as the pressure in the left ventricle and left atrium.
TMP14 pp. 1073, 1074
34. C) Consuming a meal consisting of carbohydrate, protein, and fat will
stimulate the production and release of insulin, which promotes energy
storage. Insulin increases cell permeability to glucose to promote its
storage in the form of glycogen (liver) and fat through fay acid
synthesis and storage in the adipose. Hormone-sensitive lipase
promotes the breakdown of fat to free fay acids and is decreased in
response to insulin.
TMP14 pp. 985–989
35. B) Type I diabetes is associated with low insulin and thus an impaired
ability to store energy. Thus, in the absence of insulin, plasma free fay
acids are increased to be made available for energy, liver glycogen is
depleted in an aempt to maintain plasma glucose, and skeletal muscle
mass decreases as protein is metabolized to make amino acids available
for energy.
TMP14 pp. 994–995
36. C) The postabsorptive state begins approximately 2 hours after a meal
when plasma glucose has typically returned to normal. During the
postabsorptive state, counter regulatory mechanisms are activated
which help to maintain constant plasma glucose concentration. Thus,
insulin is reduced to decrease the cellular uptake of glucose and
glucagon is increased to promote hepatic production and release of
glucose. After several hours, both growth hormone and cortisol are also
increased to reduce skeletal muscle and adipose uptake of glucose. The
net effect of these mechanisms is to prevent hypoglycemia.
TMP14 pp. 986, 991–992, 994
37. B) hCG also binds to LH receptors on the interstitial cells of the testes
of the male fetus, resulting in the production of testosterone in male
fetuses up to the time of birth. This small secretion of testosterone is
what causes the fetus to develop male sex organs instead of female sex
organs.
TMP14 pp. 1033, 1060–1061
38. D) Insulin is a peptide hormone that is derived from proinsulin. It
binds to an enzyme linked receptor composed of 2 alpha and 2 beta
subunits, leading to an increase in tyrosine kinase activity.
TMP14 pp. 984–985
39. E) The exchange of calcium between the bone fluid compartment and
the ECF serves as a rapid and fast-acting mechanism to buffer changes
in extracellular fluid calcium concentration.
TMP14 p. 1005
40. B) In nonpregnant woman, the only significant source of estrogen is
ovarian follicles or corpus luteae. Menstruation begins when the corpus
luteum degenerates. Menstruation ends when developing follicles
secrete estrogen sufficiently to raise circulating concentration to a level
that stimulates regrowth of the endometrium.
TMP14 pp. 1039, 1042, 1046–1047
41. B) As a result of negative feedback, plasma levels of TSH are a sensitive
index of circulating levels of unbound (free) thyroid hormones. High
plasma levels of TSH indicate inappropriately low levels of free thyroid
hormones in the circulation, such as are present with autoimmune
destruction of the thyroid gland in persons with Hashimoto’s disease.
However, because elevated plasma levels of estrogen in pregnancy
increase hepatic production of TBG, the total amount (bound + free) of
thyroid hormones in the circulation is elevated. Plasma levels of thyroid
hormones are elevated in persons with Graves’ disease and in patients
with a pituitary TSH-secreting tumor, as well in patients given thyroid
extract for therapy.
TMP14 pp. 954, 958–962
42. F) The kidneys are essential for the conversion of inactive vitamin D
prohormones to the biologically active vitamin D hormone (1,25-
dihydroxycholecalciferol). This conversion is mediated by parathyroid
hormone acting in the proximal tubule epithelial cells. Therefore, with
impaired renal function, one would expect a decrease in plasma [1,25-
(OH)2D], along with a compensatory increase in PTH. The increased
plasma PTH causes bone resorption of calcium.
TMP14 p. 1015
43. D) The cells of the anterior pituitary that secrete LH and FSH, along
with the cells of the hypothalamus that secrete GnRH, are inhibited by
both estrogen and testosterone. The steroids taken by the woman caused
sufficient inhibition to result in cessation of the monthly menstrual
cycle.
TMP14 pp. 1033, 1047–1048
44. D) Patients with central diabetes insipidus have an inappropriately low
secretion rate of ADH in response to changes in plasma osmolality, but
their renal response to ADH is not impaired. Because plasma levels of
ADH are depressed, the ability to concentrate urine is impaired, and a
large volume of dilute urine is excreted. Loss of water tends to increase
plasma osmolality, which stimulates the thirst center and leads to a very
high rate of water turnover.
TMP14 p. 949
45. B) NO is the vasodilator that is normally released, causing vasodilation
in these arteries.
TMP14 pp. 1027, 1034
46. B) Hydroxyapatite is the major salt found in calcified bone, and the
osteon is composed of concentric layers of calcified bone. However, an
osteocyte is a quiescent cell that resides in lacunae (spaces). Osteoblasts
are the cells that actively form new bone.
TMP14 pp. 1003, 1005–1006
47. B) One of the major physiological roles for PTH is to promote the
conversion of 25-hydroxycholecalciferol, to the active 1,25-
dihydroxycholecalciferol in the proximal tubular epithelium. The other
choices represent normal physiological actions of PTH.
TMP14 pp. 1009–1012
48. D) Lethargy and myxedema are signs of hypothyroidism. Low plasma
levels of TSH indicate that the abnormality is in either the hypothalamus
or the pituitary gland. The responsiveness of the pituitary to the
administration of TRH suggests that pituitary function is normal and
that the hypothalamus is producing insufficient amounts of TRH.
TMP14 pp. 958–962
49. D) Inhibin prevents FSH release from the anterior pituitary,
preventing Sertoli cells from causing aromatization to produce estradiol.
TMP14 p. 1032
50. A) After menopause, the absence of feedback inhibition by estrogen
and progesterone results in extremely high rates of FSH secretion.
Women taking estrogen as part of hormone replacement therapy for
symptoms associated with postmenopausal conditions have suppressed
levels of FSH as a result of the inhibitory effect of estrogen.
TMP14 pp. 1050, 1051
51. D) Phosphodiesterase-5 receptors prevent hydrolysis of cyclic
guanosine monophosphate, thus keeping the levels high and
maintaining vasodilation.
TMP14 p. 1034
52. B) Glucagon stimulates glycogenolysis in the liver, but it has no
physiological effects in muscle. Both glucagon and cortisol increase
gluconeogenesis, and cortisol impairs glucose uptake by muscle.
TMP14 pp. 972–973, 992
53. C) Injection of insulin leads to a decrease in blood glucose
concentration. Hypoglycemia stimulates the secretion of GH, glucagon,
and epinephrine, all of which have counter regulatory effects to increase
glucose levels in the blood.
TMP14 pp. 945, 993–994
54. A) Prolonged fetal hypoxia during delivery can cause serious
depression of the respiratory center. Hypoxia may occur during delivery
because of compression of the umbilical cord, premature separation of
the placenta, excessive contraction of the uterus, or excessive anesthesia
of the mother.
TMP14 p. 1073
55. C) In general, peptide hormones are water soluble and are not highly
bound by plasma proteins. ADH, a neurohypophysial peptide hormone,
is virtually unbound by plasma proteins. In contrast, steroid and thyroid
hormones are highly bound to plasma proteins.
TMP14 pp. 929–930
56. C) The rise in intracellular calcium in the oocyte triggers the cortical
reaction in which granules that previously lay at the base of the plasma
membrane undergo exocytosis. This process leads to the release of
enzymes that “harden” the zona pellucida and prevent other sperm
from penetrating.
TMP14 p. 1025
57. B) Although estrogen and progesterone are essential for the physical
development of the breast during pregnancy, a specific effect of both
these hormones is to inhibit the actual secretion of milk. Even though
prolactin levels are increased 10- to 20-fold at the end of pregnancy, the
suppressive effects of estrogen and progesterone prevent milk
production until after the baby is born. Immediately after birth, the
sudden loss of both estrogen and progesterone secretion from the
placenta allows the lactogenic effect of prolactin to promote milk
production.
TMP14 pp. 1066–1067
58. C) The concentration of PTH strongly regulates the absorption of
calcium ion from the renal tubular fluid. A reduction in hormone
concentration reduces calcium reabsorption and increases the rate of
calcium excretion in the urine. The other choices either have lile effect
on or decrease calcium excretion.
TMP14 pp. 1011–1012
59. B) A pituitary tumor secreting increased amounts of TSH would be
expected to stimulate the thyroid gland to secrete increased amounts of
thyroid hormones. TSH stimulates several steps in the synthesis of
thyroid hormones, including the synthesis of thyroglobulin. Increased
heart rate is among the many physiological responses to high plasma
levels of thyroid hormones. However, high plasma levels of thyroid
hormones do not cause exophthalmos. Immunoglobulins cause
exophthalmos in Graves’ disease, the most common form of
hyperthyroidism.
TMP14 pp. 952, 957, 961
60. A) Hemorrhage decreases the activation of stretch receptors in the atria
and arterial baroreceptors. Decreased activation of these receptors
increases ADH secretion.
TMP14 p. 949
61. E) Choices A to D are true: LH secretion will be suppressed (B) by the
negative feedback effect of the estrogen from the tumor; consequently,
she will not have menstrual cycles (C), and because she will not have
normal cycles, no corpus luteae will develop, so no progesterone will be
formed (A). The high levels of estrogen produced by the tumor will
provide stimulation of osteoblastic activity to maintain normal bone
activity (D).
TMP14 pp. 1044, 1045
62. D) After eating a meal, insulin secretion is increased. As a result, there
is an increased rate of glucose uptake by both the liver and muscle.
Insulin also inhibits hormone-sensitive lipase, which decreases
hydrolysis of triglycerides in fat cells.
TMP14 pp. 985–987, 992
63. B) The primary function of testosterone in the embryonic development
of males is to stimulate formation of the male sex organs.
TMP14 pp. 219–220, 364, 383, 405, 949–950
64. B) FSH stimulates the production of estrogens from Sertolis cells in the
testis. The Sertoli cells receive testosterone from Leydig cells (stimulated
by LH) and use the testosterone to make estrogen.
TMP14 p. 1023
65. C) The reduction in hydrogen ion indicated by the elevation in pH
increases the concentration of negatively charged phosphate ion species
available for ionic combination with calcium ions. Consequently, the
free calcium ion concentration is reduced.
TMP14 pp. 1011–1012
66. A) Prostate fluid contains calcium, citrate, phosphate and fibrinolysin.
The function of prostate fluid is to help neutralize the acidic
environment associated with other seminal fluids and thus improve
sperm motility.
TMP14 p. 1024
67. C) During suckling, stimulation of receptors on the nipples increases
neural input to both the supraoptic and paraventricular nuclei.
Activation of these nuclei leads to the release of oxytocin and
neurophysin from secretion granules in the posterior pituitary gland.
Suckling does not stimulate the secretion of appreciable amounts of
ADH.
TMP14 pp. 1066, 1067
68. C) In Conn’s syndrome, large amounts of aldosterone are secreted.
Because aldosterone causes sodium retention, hypertension is a
common finding in patients with this condition. However, the degree of
sodium retention is modest, as is the resultant increase in extracellular
fluid volume. This occurs because the rise in arterial pressure offsets the
sodium-retaining effects of aldosterone, limiting sodium retention and
permiing daily sodium balance to be achieved.
TMP14 pp. 970, 981
69. C) The activity of stored sperm is aenuated as a result of the acidic
environment. After ejaculation, uterine and fallopian fluids wash away
inhibitory factors, allowing for full activation of the spermatozoa.
TMP14 pp. 1024–1025
70. D) DHEA sulfate produced by the fetal adrenal gland diffuses to the
placenta and is converted to DHEA and then to estradiol and provides
estradiol to the mother.
TMP14 pp. 1060, 1061
71. D) Sporadic nursing of the mother results in a lack of prolactin surge
because mechanosensors in the nipple cause prolactin release. Without
prolactin release, there is a lack of milk production, and the mother
eventually will not be able to provide milk for the baby.
TMP14 pp. 1066, 1067
72. A) Persons with Addison’s disease have diminished secretion of both
glucocorticoids (cortisol) and mineralocorticoids (aldosterone). In
persons with Cushing’s disease or Cushing’s syndrome, cortisol
secretion is elevated, but aldosterone secretion is normal. A low-sodium
diet is associated with a high rate of aldosterone secretion but a
secretion rate of cortisol that is normal. By inhibiting the generation of
angiotensin II and thus the stimulatory effects of angiotensin II on the
zona glomerulosa, administration of a converting enzyme inhibitor
would decrease aldosterone secretion without altering the rate of
cortisol secretion.
TMP14 pp. 971–972, 979–980
73. E) Spermatogonia undergo two rounds of meiotic division, leading to
the production of four haploid spermatids. The spermatids ultimately
differentiate into mature sperm.
TMP14 pp. 1021–1022
74. B) Progesterone is required to maintain the decidual cells of the
endometrium. If progesterone levels fall, as they do during the last days
of a nonpregnant menstrual cycle, menstruation will follow within a few
days, with loss of pregnancy. Administration of a compound that blocks
the progesterone receptor during the first few days after conception will
terminate the pregnancy.
TMP14 pp. 1060–1061
75. D) An inappropriately high rate of ADH secretion from the lung
promotes excess water reabsorption, which tends to produce
concentrated urine and a decrease in plasma osmolality. Low plasma
osmolality suppresses both thirst and ADH secretion from the pituitary
gland.
TMP14 pp. 404, 949
76. B) A very high plasma concentration of progesterone maintains the
uterine muscle in a quiescent state during pregnancy. In the final month
of gestation, the concentration of progesterone begins to decline,
increasing the excitability of the muscle.
TMP14 pp. 971–972, 1027
77. D) The corpus luteum is the only source of progesterone. If she is not
having menstrual cycles, no corpus luteum is present.
TMP14 p. 1048
78. C) FSH stimulates the granulosa cells of the follicle to secrete estrogen.
TMP14 pp. 1040, 1048
79. E) In response to increased blood levels of glucose, plasma insulin
concentration normally increases during the 60-minute period following
oral intake of glucose. In type 1 diabetes mellitus, insulin secretion is
depressed. In contrast, in type 2 diabetes mellitus, insulin resistance is a
common finding, and at least in the early stages of the disease, there is
an abnormally high rate of insulin secretion.
TMP14 pp. 995–998
80. D) In Cushing’s syndrome, high plasma levels of cortisol impair
glucose uptake in peripheral tissues, which tends to increase plasma
levels of glucose. As a result, the insulin response to oral intake of
glucose is enhanced.
TMP14 pp. 996–998
81. B) In general, protein hormones cause physiological effects by binding
to receptors on the cell membrane. However, of the four protein
hormones indicated, only insulin activates an enzyme-linked receptor.
Aldosterone is a steroid hormone and enters the cytoplasm of the cell
before binding to its receptor.
TMP14 p. 932
82. D) hCG is secreted from the trophoblast cells beginning shortly after
the blastocyst implants in the endometrium.
TMP14 pp. 1060–1061
83. B) Aortic pressure increases due to the increase in left ventricular
pressure. The increase in left atrial pressure causes the foramen ovale to
close. The ductus arteriosus also closes within a short time after birth.
TMP14 pp. 1073–1075
84. A) Somnolence is a common feature of hypothyroidism. Palpitations,
increased respiratory rate, increased cardiac output, and weight loss are
all associated with hyperthyroidism.
TMP14 pp. 957, 962–963
85. C) An infant born of a mother with untreated diabetes will have
considerable hypertrophy and hyperfunction of the islets of Langerhans
in the pancreas. As a consequence, the infant’s blood glucose
concentration may fall to lower than 20 mg/dl shortly after birth.
TMP14 pp. 1078–1079
86. B) If a successful fertilization event occurs, followed by implantation in
the uterine wall, trophoblasts produce and secrete human chorionic
gonadotropin, which maintains the corpus luteum and its production of
estrogen and progesterone. Eventually, hCG levels decline in association
with increased placental production of progesterone and estrogen.
TMP14 p. 1042
87. E) Choices A to D would not stimulate PTH secretion. An increase in
calcium concentration (A) suppresses PTH secretion; calcitonin has lile
to no effect on PTH secretion (B); acidosis would increase free calcium in
the extracellular fluid, thereby inhibiting PTH secretion (C); and PTHreleasing hormone does not exist (D).
TMP14 pp. 1001, 1011
88. C) Potassium is a potent stimulus for aldosterone secretion, as is
angiotensin II. Therefore, a patient consuming a high-potassium diet
would exhibit high circulating levels of aldosterone.
TMP14 p. 971
89. B) The decidua and trophoblasts provide the nutrition needed to
provide nourishment of the blastocyst.
TMP14 pp. 1057, 1060–1062
90. C) Steroid hormones are not stored to any appreciable extent in their
endocrine-producing glands. This is true for aldosterone, which is
produced in the adrenal cortex. In contrast, there are appreciable stores
of thyroid hormones and peptide hormones in their endocrineproducing glands.
TMP14 p. 928
91. C) 1,25-Dihydroxycholecalciferol is formed only in the renal cortex.
Extensive renal disease reduces the amount of cortical tissue,
eliminating the source of this active calcium regulating hormone.
TMP14 p. 1015
92. C) The placenta cannot produce androgens but can only produce
DHEA by removal of the sulfate from DHEAS produced in the fetal
adrenal glands.
TMP14 p. 1060
93. A) The secretory phase of the endometrial cycle aligns with the luteal
phase of the ovarian cycle. Progesterone levels peak during this phase
and promote the vascularization and thickening of the endometrial
lining. If a fertilization event and subsequent implantation does not
occur, the corpus luteum involutes causing progesterone levels to fall
and the endometrial lining to slough off during menstruation.
TMP14 pp. 1046–1047
94. D) Because iodine is needed to synthesize thyroid hormones, the
production of thyroid hormones is impaired if iodine is deficient. As a
result of feedback, plasma levels of TSH increase and stimulate the
follicular cells to increase the synthesis of thyroglobulin, which results
in a goiter. Increased metabolic rate, sweating, nervousness, and
tachycardia are all common features of hyperthyroidism, not
hypothyroidism, due to iodine deficiency.
TMP14 pp. 960–963
95. C) Because of the effects of thyroid hormones to increase metabolism in
tissues, tissues vasodilate, thus increasing blood flow and cardiac
output. All the other choices increase in response to high plasma levels
of thyroid hormones.
TMP14 pp. 956–957
96. B) Sperm cell motility decreases as pH is reduced below 6.8. At a pH of
4.5, sperm cell motility is significantly reduced. However, the buffering
effect of sodium bicarbonate in the prostatic fluid raises the pH
somewhat, allowing the sperm cells to regain some mobility.
TMP14 p. 1024
97. B) A protein meal stimulates all three hormones indicated.
TMP14 pp. 945, 991, 993
98. C) Testosterone secreted by the testes in response to LH inhibits
hypothalamic secretion of GnRH, thereby inhibiting anterior pituitary
secretion of LH and FSH. Taking large doses of testosterone-like steroids
also suppresses the secretion of GnRH and the pituitary gonadotropic
hormones, resulting in sterility.
TMP14 p. 1033
99. C) Steroids with potent glucocorticoid activity tend to increase plasma
glucose concentration. As a result, insulin secretion is stimulated.
Increased glucocorticoid activity also diminishes muscle protein.
Because of feedback, cortisone administration leads to a decrease in
adrenocorticotropic hormone secretion and therefore a decrease in
plasma cortisol concentration.
TMP14 pp. 972–973
100. C) Inhibin is the hormone that has a negative feedback on the anterior
pituitary to prevent FSH from being released. Inhibin is produced by the
granulosa cells in the ovary.
TMP14 pp. 1040–1041
101. A) An increase in the concentration of PTH results in the stimulation
of existing osteoclasts and, over longer periods, increases the number of
osteoclasts present in the bone.
TMP14 pp. 1010–1011
102. B) In general, peptide hormones produce biological effects by binding
to receptors on the cell membrane. Peptide hormones are stored in
secretion granules in their endocrine-producing cells and have relatively
short half-lives because they are not highly bound to plasma proteins.
Protein hormones often have a rapid onset of action because, unlike
steroid and thyroid hormones, protein synthesis is usually not a
prerequisite to produce biological effects.
TMP14 pp. 926, 929–932
103. D) A pituitary tumor secreting GH is likely to present as an increase in
pituitary gland size. The anabolic effects of excess GH secretion lead to
enlargement of the internal organs, including the kidneys. Because
acromegaly is the state of excess GH secretion after epiphyseal closure,
increased femur length does not occur.
TMP14 p. 947
104. A) GH and cortisol have opposite effects on protein synthesis in
muscle. GH is anabolic and promotes protein synthesis in most cells of
the body, whereas cortisol decreases protein synthesis in extrahepatic
cells, including muscle. Both hormones impair glucose uptake in
peripheral tissues and therefore tend to increase plasma glucose
concentration. Both hormones also mobilize triglycerides from fat stores.
TMP14 pp. 943–944, 972–973
105. B) If the mother has had adequate amounts of iron in her diet, the
infant’s liver usually has enough stored iron to form blood cells for 4 to
6 months after birth. However, if the mother had insufficient iron levels,
severe anemia may develop in the infant after about 3 months of life.
TMP14 pp. 1072, 1077
106. A) High plasma levels of steroids with glucocorticoid activity
suppress CRH and, consequently, ACTH secretion. Therefore, the
adrenal glands would actually atrophy with chronic cortisone treatment.
Increased plasma levels of glucocorticoids tend to cause sodium
retention and increase blood pressure. They also tend to increase plasma
levels of glucose and, consequently, stimulate insulin secretion and Cpeptide, which is part of the insulin prohormone.
TMP14 pp. 972–973, 976–977, 979–980
107. B) During the postovulatory phase of the cycle, there is a negative
feedback relationship between progesterone and estrogen and the
hypothalamic pituitary axis. Therefore, progesterone suppresses GnRH
release.
TMP14 pp. 1040–1042
108. C) SRY is the region on the Y chromosome that encodes a
transcription factor that causes differentiation of Sertoli cells from
precursors in testis. If SRY is not present, granulosa cells in the ovary are
produced.
TMP14 p. 1029
109. D) Fertilization of the ovum normally takes place in the ampulla of
one of the fallopian tubes.
TMP14 p. 1055
110. D) Because insulin secretion is deficient in persons with type 1
diabetes mellitus, there is increased (not decreased) release of glucose
from the liver. Low plasma levels of insulin also lead to a high rate of
lipolysis; increased plasma osmolality, hypovolemia, and acidosis are all
symptoms of uncontrolled type 1 diabetes mellitus.
TMP14 pp. 995–996
111. E) Under acute conditions, an increase in blood glucose concentration
will decrease GH secretion. GH secretion is characteristically elevated in
the chronic pathophysiological states of acromegaly and gigantism.
Deep sleep and exercise are stimuli that increase GH secretion.
TMP14 pp. 945–946
112. D) All the steroids listed include pregnenolone early in their
biosynthetic pathway. 1,25(OH)2D is derived from vitamin D and does
not include pregnenolone in its biosynthetic pathway.
TMP14 pp. 965–967, 1007–1008
113. D) Estrogen and, to a lesser extent, progesterone secreted by the
corpus luteum during the luteal phase have strong feedback effects on
the anterior pituitary gland to maintain low secretory rates of both FSH
and LH. In addition, the corpus luteum secretes inhibin, which inhibits
the secretion of FSH.
TMP14 p. 1042
114. D) Under chronic conditions, the effects of high plasma levels of
aldosterone to promote sodium reabsorption in the collecting tubules
are sustained. However, persistent sodium retention does not occur
because of concomitant changes that promote sodium excretion. These
changes include increased arterial pressure, increased plasma levels of
g p p
atrial natriuretic peptide, and decreased plasma angiotensin II
concentration.
TMP14 pp. 961, 981
115. B) For reasons that are not entirely clear, the negative feedback
regulation between estrogen and LH that occurs throughout the ovarian
cycle briefly changes to a positive feedback mechanism. This occurs late
in the follicular phase, just prior to ovulation, when LH promotes
estrogen production and estrogen feeds back to stimulate the further
release of LH. This underlies the surge in LH just before ovulation.
TMP14 pp. 1040–1042
116. B) Circulating levels of free T4
exert biological effects and are
regulated by feedback inhibition of TSH secretion from the anterior
pituitary gland. Protein-bound T4
is biologically inactive. Circulating T4
is highly bound to plasma proteins, especially to TBG, which increases
during pregnancy. An increase in TBG tends to decrease free T4
, which
then leads to an increase in TSH secretion, causing the thyroid to
increase thyroid hormone secretion. Increased secretion of thyroid
hormones persists until free T4
returns to normal levels, at which time
there is no longer a stimulus for increased TSH secretion. Therefore, in a
chronic steady-state condition associated with elevated TBG, high
plasma total T4
(bound and free) and normal plasma TSH levels would
be expected. In this pregnant patient, the normal levels of total T4
, along
with high plasma levels of TSH, would indicate an inappropriately low
plasma level of free T4
. Deficient thyroid hormone secretion in this
patient would be consistent with Hashimoto’s disease, the most
common form of hypothyroidism.
TMP14 pp. 954, 958–962
117. D) The motor neurons of the spinal cord of the thoracic and lumbar
regions are the sources of innervation for the skeletal muscles of the
perineum involved in ejaculation.
TMP14 pp. 1026, 1027
118. A) Trophoblasts invade the endometrial lining of the uterus and
provide nutrients to the growing blastocyst until the placenta if formed.
TMP14 pp. 1056–1057
119. B) Bone is deposited in proportion to the compressional load that the
bone must carry. Continual mechanical stress stimulates osteoblastic
deposition and calcification of bone.
TMP14 pp. 1006–1007
120. D) Prolactin is produced in the anterior pituitary from lactotrope cells
and is responsible for promoting milk production and secretion.
TMP14 pp. 1067–1068
121. B) In the absence of 11-β-hydroxysteroid dehydrogenase, renal
epithelial cells cannot convert cortisol to cortisone; therefore, cortisol
will bind to the mineralocorticoid receptor and mimic the actions of
excess aldosterone. Consequently, this would result in hypertension
associated with suppression of the renin-angiotensin-aldosterone
system, along with hypokalemia.
TMP14 pp. 968–970, 980–981
122. D) In target tissues, nuclear receptors for thyroid hormones have a
greater affinity for T3
than for T4
. The secretion rate, plasma
concentration, half-life, and onset of action are all greater for T4
than for
T3
.
TMP14 pp. 953–955
123. C) Blocking the action of FSH on the Sertoli cells of the seminiferous
tubules interrupts the production of sperm. Choice C is the only option
that is certain to provide sterility.
TMP14 p. 1033
124. C) Oxytocin is secreted from the posterior pituitary gland and carried
in the blood to the breast, where it causes the cells that surround the
outer walls of the alveoli and ductile system to contract. Contraction of
these cells raises the hydrostatic pressure of the milk in the ducts to 10 to
20 mm Hg. Consequently, milk flows from the nipple into the baby’s
mouth.
TMP14 pp. 1068–1069
125. B) Resulting from the growing fetal-placental unit, there is a large
increase in metabolic demand during a normal pregnancy. Given that
metabolic demand is the major determinant for cardiac output, the
increase in metabolic demand during pregnancy causes an increase in
cardiac output.
TMP14 p. 1062
126. F) Persons with Cushing’s disease have a high rate of cortisol
secretion, but aldosterone secretion is normal. High plasma levels of
cortisol tend to increase plasma glucose concentration by impairing
glucose uptake in peripheral tissues and by promoting gluconeogenesis.
However, at least in the early stages of Cushing’s disease, the tendency
for glucose concentration to increase appreciably is counteracted by
increased insulin secretion.
TMP14 pp. 972–973, 979–980
127. A) In healthy patients, the secretory rates of ACTH and cortisol are
low in the late evening but high in the early morning. In patients with
Cushing’s syndrome (adrenal adenoma) or in patients taking
dexamethasone, plasma levels of ACTH are very low and are certainly
not higher than normal early morning values. In patients with
Addison’s disease, plasma levels of ACTH are elevated as a result of
deficient adrenal secretion of cortisol. The secretion of ACTH and
cortisol would be expected to be normal in Conn’s syndrome.
TMP14 pp. 977–980
128. B) Exercise stimulates GH secretion. Hyperglycemia, somatomedin,
and the hypothalamic inhibitory hormone somatostatin all inhibit GH
secretion. GH secretion also decreases as persons age.
TMP14 p. 945
129. C) A low-sodium diet would stimulate aldosterone but not cortisol
secretion. Increased atrial stretch associated with volume expansion
would stimulate atrial natriuretic peptide secretion but would not be
expected during a low-sodium diet.
TMP14 pp. 364, 405, 971–972
130. A) Adrenal gland hypofunction with Addison’s disease is associated
with decreased secretion of both aldosterone and cortisol. In Cushing’s
disease and Cushing’s syndrome associated with an ectopic tumor, the
mineralocorticoid-hypertension induced by high plasma levels of
cortisol would suppress aldosterone secretion. Neither a high-sodium
diet nor administration of a converting enzyme inhibitor would affect
cortisol secretion.
TMP14 pp. 971-972, 979–980
131. B) Blood returning from the placenta through the umbilical vein
passes through the ductus venosus. The blood coming from the placenta
has the highest concentration of oxygen found in the fetus.
TMP14 p. 1074
132. B) Osteoporosis, hypertension, hirsutism, and hyperpigmentation are
all symptoms of Cushing’s syndrome associated with high plasma levels
of ACTH. If the high plasma ACTH levels were the result of either a
pituitary adenoma or an abnormally high rate of corticotropin-releasing
hormone secretion from the hypothalamus, the patient would likely
have an enlarged pituitary gland. In contrast, the pituitary gland would
not be enlarged if an ectopic tumor were secreting high levels of ACTH.
TMP14 pp. 979–980
133. B) Prolactin secretion is inhibited, not stimulated, by the hypothalamic
release of dopamine into the median eminence. GH is inhibited by the
hypothalamic-inhibiting hormone somatostatin. The secretion of LH,
TSH, and ACTH are all under the control of the releasing hormones
indicated.
TMP14 p. 942
134. B) Increased heart rate, increased respiratory rate, and decreased
cholesterol concentration are all responses to excess thyroid hormone.
TMP14 pp. 956–958
135. C) hCG is produced by syncytial trophoblasts from the growing
blastocyst. hCG is responsible for maintaining the corpus luteum which
produces estrogens and progesterone up through approximately 12
weeks’ gestation. After that time, the placenta makes enough estrogen
and progesterone to sustain the pregnancy.
TMP14 pp. 1059–1060
136. D) By age 45 years, only a few primordial follicles remain in the
ovaries to be stimulated by gonadotropic hormones, and the production
of estrogen decreases as the number of follicles approaches zero. When
estrogen production falls below a critical value, it can no longer inhibit
g p g
the production of gonadotropic hormones from the anterior pituitary.
FSH and LH are produced in large quantities, but as the remaining
follicles become atretic, production by the ovaries falls to zero.
TMP14 pp. 1050, 1051
137. D) The binding of insulin to its receptor activates tyrosine kinase,
resulting in metabolic events leading to increased synthesis of fats,
proteins, and glycogen. In contrast, gluconeogenesis is inhibited.
TMP14 pp. 984–989
138. C) The secretion of chemical messengers (neurohormones) from
neurons into the blood is referred to as neuroendocrine secretion. Thus,
in contrast to the local actions of neurotransmiers at nerve endings,
neurohormones circulate in the blood before producing biological
effects at target tissues. Oxytocin is synthesized from magnocellular
neurons whose cell bodies are located in the paraventricular and
supraoptic nuclei and whose nerve terminals terminate in the posterior
pituitary gland. Target tissues for circulating oxytocin are the breast and
uterus, where the hormone plays a role in lactation and parturition,
respectively.
TMP14 pp. 925, 948–950
139. C) The placenta is hypoxic under normal physiological conditions.
The diffusion of oxygen from the maternal circulation to the fetal
circulation is enhanced by the fact that fetal hemoglobin carries a greater
quantity of oxygen at a given blood Po2 than maternal hemoglobin. In
addition, the hemoglobin concentration is greater in the fetal circulation
than in the maternal circulation.
TMP14 p. 1058
140. B) Inhibition of the iodide pump decreases the synthesis of thyroid
hormones but does not impair the production of thyroglobulin by
follicular cells. Decreased plasma levels of thyroid hormones result in a
low metabolic rate and lead to an increase in TSH secretion. Increased
plasma levels of TSH stimulate the follicular cells to synthesize more
thyroglobulin. Nervousness is a symptom of hyperthyroidism and is not
caused by thyroid hormone deficiency.
TMP14 pp. 951–952, 956–960
141. D) As the blastocyst implants, the trophoblast cells invade the
decidua, digesting and imbibing it. The stored nutrients in the decidual
cells are used by the embryo for growth and development. During the
first week after implantation, this is the only means by which the
embryo can obtain nutrients. The embryo continues to obtain at least
some of its nutrition in this way for up to 8 weeks, although the placenta
begins to provide nutrition after about the 16th day beyond fertilization
(a lile more than 1 week after implantation).
TMP14 p. 1056
142. A) Both ADH and oxytocin are peptides containing nine amino acids.
Their chemical structures differ in only two amino acids.
TMP14 p. 949
143. A) Because glucocorticoids decrease the sensitivity of tissues to the
metabolic effects insulin, they would exacerbate diabetes.
Thiazolidinediones and weight loss increase insulin sensitivity.
Sulfonylureas increase insulin secretion. If weight loss and the
aforementioned drugs are ineffective, exogenous insulin may be used to
regulate blood glucose concentration.
TMP14 pp. 991, 996–997
144. C) In the early stages of type 2 diabetes, the tissues have a decreased
sensitivity to insulin. As a result, there is a tendency for plasma glucose
to increase, in part because decreased hepatic insulin sensitivity leads to
increased hepatic glucose output. Because of the tendency for plasma
glucose to increase, there is a compensatory increase in insulin secretion,
including C-peptide, which is part of the insulin prohormone.
Hypovolemia and increased production of ketone bodies, although
commonly associated with uncontrolled type 1 diabetes, are not
typically present in the early stages of type 2 diabetes.
TMP14 pp. 984, 994–998
145. C) One of the most characteristic findings in respiratory distress
syndrome is failure of the respiratory epithelium to secrete adequate
quantities of surfactant into the alveoli. Surfactant decreases the surface
tension of the alveolar fluid, allowing the alveoli to open easily during
inspiration. Without sufficient surfactant, the alveoli tend to collapse,
and there is a tendency to develop pulmonary edema.
TMP14 p. 1074
146. D) Several circulatory changes occur in the fetal circulation after birth.
These include the closing of physiological shunts. The ductus arteriosus
is a shunt that carries blood from the fetal pulmonary artery into the
descending aorta, thus bypassing the pulmonary circulation. At birth,
this shunt closes as systemic resistance increases, causing blood to flow
back into the pulmonary circulation through the shunt. Within hours of
birth, the walls of the ductus arteriosus close, and eventually the closing
becomes fibrous for permanent closure.
TMP14 pp. 1074–1075
147. C) The primary controllers of ACTH, GH, LH, and TSH secretion from
the pituitary gland are hypothalamic-releasing hormones. They are
secreted into the median eminence and subsequently flow into the
hypothalamic-hypophysial portal vessels before bathing the cells of the
anterior pituitary gland. Conversely, prolactin secretion from the
pituitary gland is influenced primarily by the hypothalamic-inhibiting
hormone dopamine. Consequently, obstruction of blood flow through
the portal vessels would lead to reduced secretion of ACTH, GH, LH,
and TSH but increased secretion of prolactin.
TMP14 p. 942
148. D) Osteoblasts secrete all of these except pyrophosphate. Secretions
(alkaline phosphatase) from osteoblasts neutralize pyrophosphate, an
inhibitor of hydroxyapatite crystallization. Neutralization of
pyrophosphate permits the precipitation of calcium salts into collagen
fibers.
TMP14 pp. 1004–1006
149. B) In primary hyperparathyroidism, high plasma levels of PTH
increase the formation of 1,25-(OH)2D3, which increases intestinal
absorption of calcium. This action of PTH, along with its effects to
increase bone resorption and renal calcium reabsorption, leads to
hypercalcemia. However, because of the high filtered load of calcium,
calcium is excreted in the urine. High plasma levels of PTH also
decrease phosphate reabsorption and increase urinary excretion, leading
to a fall in plasma phosphate concentration.
TMP14 pp. 1009–1012, 1014–1015
150. A) Gamma radiation destroys the cells undergoing the most rapid
rates of mitosis and meiosis, the germinal epithelium of the testes. The
man described is said to have normal testosterone levels, suggesting that
the secretory paerns of GnRH and LH are normal and that his
interstitial cells are functional. Because he is not producing sperm, the
levels of inhibin secreted by the Sertoli cells would be maximally
suppressed, and his levels of FSH would be strongly elevated.
TMP14 p. 1033
151. B) In this experiment, the size of the thyroid gland increased because
TSH causes hypertrophy and hyperplasia of its target gland and
increased secretion of thyroid hormones. Increased plasma levels of
thyroid hormones inhibit the secretion of TRH, which decreases
stimulation of the pituitary thyrotropes, resulting in a decrease in the
size of the pituitary gland. Higher plasma levels of thyroid hormones
also increase metabolic rate and decrease body weight.
TMP14 pp. 955–955, 960
152. C) In this experiment, the size of the pituitary and adrenal glands
increased because CRH stimulates the pituitary corticotropes to secrete
ACTH, which in turn stimulates the adrenals to secrete corticosterone
and cortisol. Higher plasma levels of cortisol increase protein
degradation and lipolysis and therefore decrease body weight.
TMP14 pp. 972–974, 976–977
153. C) At birth, the neonatal liver is not fully functional. Therefore, it does
not excrete bilirubin properly over the first several days of life. The
increased concentration of circulating bilirubin gives infants a yellow
pigmentation in the skin and eyes (jaundice).
TMP14 pp. 1076–1077
Unit XV: Sports Physiology
1. A Tour de France rider has the following values under resting
conditions:
Oxygen consumption = 250 ml O2
/min
Hemoglobin concentration = 15 g Hg/dl
Arterial partial pressure of oxygen (PO 2
) = 100 mm Hg
Mixed venous saturation = 75 percent
When exercising, he has the following values:
Oxygen consumption = 3000 ml O2
/min
Hemoglobin concentration = 15 g/dl
Arterial PO 2
= 100 mm Hg
Mixed venous saturation = 25%
What is the absolute increase in cardiac output with exercise?
A) 5 l/min
B) 15 l/min
C) 25 l/min
D) 30 l/min
2. A female university student is comfortably running a 10K
race. At 5 miles, which set of values would best describe her
blood composition?
Arterial PO 2 Arterial PCO 2 Mixed Venous PO 2
A) ↑ ↑ ↓
B) ↑ ↑ ↔
C) ↑ ↓ ↔
D) ↑ ↔ ↓
E) ↑ ↔ ↑
F) ↔ ↔ ↔
G) ↓ ↑ ↓
H) ↓ ↓ ↓
I) ↓ ↑ ↔
3. Which statement about respiration in exercise is most
accurate?
A) Maximum oxygen consumption of a male marathon
runner is less than that of an untrained average man
B) Maximum oxygen consumption can be increased about
100% by training
C) Maximum oxygen diffusing capacity of a male
marathon runner is much greater than that of an
untrained average man
D) Blood levels of oxygen and carbon dioxide are
abnormal during exercise
4. Olympic athletes who run marathons or cross-country ski
have much higher maximum cardiac outputs than
nonathletes. Which statement about the hearts of these
athletes compared with nonathletes is most accurate?
A) Stroke volume in the Olympic athletes is about 5%
greater at rest
B) The percentage increase in heart rate during maximal
exercise is much greater in the Olympic athletes
C) Maximum cardiac output is only 3% to 4% greater in the
Olympic athletes
D) Resting heart rate in the Olympic athletes is
significantly higher
5. In athletes who use androgens to increase performance,
which of the following would most likely occur?
A) Decreased high-density blood lipoproteins
B) Decreased low-density blood lipoproteins
C) Increased testicular function
D) Decreased incidence of hypertension
6. A person living in Maine trains regularly to run 10K races
and continually finishes in the middle of the pack. What is
the physiological limitation that prevents this person from
improving?
A) Limited ability to increase pulmonary ventilation
B) Limited ability to use the oxygen delivered to the tissue
C) Limited ability to increase cardiac output
D) Limited ability to dissipate the heat generated with
exercise
E) Limited ability to convert glucose to adenosine
triphosphate (ATP)
7. If muscle strength is increased with resistive training, which
condition will most likely occur?
A) A decrease in the number of myofibrils
B) An increase in mitochondrial enzymes
C) A decrease in the components of the phosphagen
energy system
D) A decrease in stored triglycerides
8. Which of the following normally occurs during exercise at
aerobic levels?
A) Increased arterial PCO 2
B) Increased alveolar PO 2
C) Increased alveolar-arterial PO 2
gradient
D) Decreased alveolar-arterial Po2
gradient
9. A 40-year-old man is performing a maximum oxygen
consumption test. He is 4 minutes into this 15-minute test.
Which of the following would best describe his blood
composition?
Arterial Po2 Arterial Pco2 Mixed venous Po2
A) ↓ ↔ ↓
B) ↓ ↑ ↓
C) ↑ ↑ ↑
D) ↑ ↓ ↑
E) ↑ ↑ ↔
F) ↑ ↓ ↓
G) ↑ ↔ ↑
H) ↓ ↓ ↓
10. Which of the following physiologic responses to acute
exercise do not normally occur?
A) Decreased blood flow to bone and gastrointestinal tract.
B) Increased cardiac output.
C) Increased minute ventilation
D) Decrease in sympathetic stimulation
Answers
1. B) At rest:
Arterial content (Ca) = 15 × 1.34 = 20 ml O2
/100 ml blood at 100%
saturation
Venous content (Cv) = 20 × 0.75 = 15 ml O2
/100 ml blood
Arteriovenous O2
difference = 5 ml O2
/100 ml blood
Answer:
VO 2
= Q (ml/min) (Ca – Cv) 250 ml O2
/min = Q (5 ml O2
/100 ml
blood)
Q = 250 ml O2
/min ÷ 5 ml O2
/100 ml blood
Q = 5.0 l/min
Exercising:
Arterial content (Ca) = 15 × 1.34 = 20 ml O2
/100 ml blood
Venous content (Cv) = 20 × 0.25 = 5 ml O2
/100 ml blood
Arteriovenous O2
difference = 15 ml O2
/100 ml blood
Answer:
VO 2
= Q (ml/min) (Ca – Cv) 3000 ml O2
/min = Q (15 ml O2
/100 ml
blood)
Q = 3000 ml O2
/min ÷ 15 ml O2
/100 ml blood
Q = 20 l/min
The increase in VO 2
is 20 l/min − 5 l/min = 15 l/min.
TMP14 pp. 256, 525, 1079–1081
2. D) With exercise, an increase in arterial PO 2
occurs as a result of
beer ventilation/perfusion. Arterial PCO 2
may be normal or
slightly decreased. Because of the increased metabolic rate, the
venous PO 2
will decrease.
TMP14 pp. 1079–1081
3. C) During exercise, the maximum oxygen consumption of a male
marathon runner is much greater than that of an untrained
average man. However, athletic training increases the maximum
oxygen consumption by only about 10%. Therefore, the maximum
oxygen consumption in marathon runners is probably partly
genetically determined. These runners also have a large increase in
maximum oxygen diffusing capacity, and their blood levels of
oxygen and carbon dioxide remain relatively normal during
exercise.
TMP14 p. 1079
4. B) When comparing Olympic athletes and nonathletes, there are
several differences in the responses of the heart. Stroke volume is
much higher at rest in an Olympic athlete, and heart rate is much
lower. The heart rate can increase approximately 270% in an
Olympic athlete during maximal exercise, which is a much greater
percentage than occurs in a nonathlete. In addition, the maximal
increase in cardiac output is approximately 30% greater in the
Olympic athlete.
TMP14 pp. 1080–1081
5. A) Use of male sex hormones (androgens) or other anabolic
steroids to increase muscle strength may increase athletic
performances under some conditions but can have adverse effects
on the body. Anabolic steroids increase the risk of cardiovascular
damage because they increase the prevalence of hypertension,
decrease high-density blood lipoproteins, and increase lowdensity blood lipoproteins. These factors all promote heart aacks
and strokes. These androgenic substances also decrease testicular
function, which decreases the formation of sperm and the body’s
own production of natural testosterone.
TMP14 p. 1083
6. C) Pulmonary ventilation is not a limitation because people
normally overventilate during exercise, and there are minimal
changes in arterial blood gases. The muscles use the oxygen
delivered to them. The limitation is the delivery of oxygen and
nutrients to muscle based on the limitation of an increase in
cardiac output. Increasing cardiac output increases exercise
performance. Under hot conditions, heat dissipation can limit
exercise performance. Muscles have minimal to no limitation in
converting glucose to ATP.
TMP14 pp. 1078–1081
7. B) During resistive training, the muscles that are contracted with
at least a 50% maximal force for at least three times a week
experience an optimal increase in muscle strength. This increase in
strength causes muscle hypertrophy, and several changes occur.
There will be an increase in the number of myofibrils and up to a
120% increase in mitochondrial enzymes. As much as a 60% to
80% increase in the components of the phosphagen energy system
can occur, and up to a 50% increase in stored glycogen can occur.
Also, as much as a 75% to 100% increase in stored triglycerides can
occur.
TMP14 p. 1078
8. C) During exercise at aerobic levels, the increase in respiration
occurs to maintain a normal alveolar Po2
, maybe a slight increase.
There is no increase in arterial Pco2
. With normal ventilation, there
would be no decrease in the alveolar–arterial Po2
gradient, which
would actually decrease oxygen diffusion from the alveoli into the
blood. With a decreased venous Po2 there will be an increased
alveolar-arterial Po2 gradient.
TMP14 pp. 517, 1080
9. F) Arterial Po2
will increase because of a beer V/Q match. Arterial
Pco2
will be normal or decreased. Mixed venous Po2
will decrease.
TMP14 pp. 1079–1080
10. D) During exercise, there are increases in blood flow to muscle
and decrease in blood flow to nonessential tissues. This is
accomplished by an increase in sympathetic stimulation. There are
also increases in cardiac output, ventilation, and cardiac
contractility.
TMP14 pp. 260, 796, 1079–1081
Normal Values for Selected
Common Laboratory
Measurements
Substance
Average
(“Normal”
Value)
Range Comment/Unit of Measure
Electrolytes
Sodium (Na
+
)
142 mmol/l 135-145
mmol/l
mmol/l = millimoles per liter
Potassium (K+
)
4.2 mmol/l 3.5-5.3
mmol/l
Chloride (Cl
–
)
106 mmol/l 98-108
mmol/l
Anion gap
12 mEq/l 7-16
mEq/l
mEq/l = milliequivalents per liter
Anion gap = Na
+ – Cl
– – HCO3−
Bicarbonate (HCO3
−
)
24 mmol/l 22-29
mmol/l
Hydrogen ion (H+
)
40 nmol/l 30-50
nmol/l
nmol/l = nanomoles per liter
pH, arterial
7.4 7.25-7.45
pH, venous
7.37 7.32-7.42
Calcium ion (Ca
2+
)
5.0 mg/dl 4.65-5.28
mg/dl
mg/dl = milligrams/deciliter
Average normal value can also be
expressed as ∼ 1.2 mmol/l or 2.4 mEq/l
Calcium, total
10.0 mg/dl 8.5-10.5
mg/dl
Magnesium ion (Mg
2+
)
0.8 mEq/l 0.6-1.1
mEq/l
Magnesium, total
1.8 mEq/l 1.3-2.4
mEq/l
Phosphate, total
3.5 mg/dl 2.5-4.5
mg/dl
In plasma, HPO4
=
is ∼1.05 mmol/l and
H2PO4
–
is 0.26 mmol/l
Nonelectrolyte Blood Chemistries
Albumin
4.5 g/dl 3.5-5.5
g/dl
g/dl = grams per deciliter
Alkaline phosphatase
M: 38-
126 U/l
U/l = units per liter
Substance
Average
(“Normal”
Value)
Range Comment/Unit of Measure
F: 70-230
U/l
Bilirubin, total
0.2-1.0
mg/dl
Bilirubin, conjugated
0-0.2
mg/dl
Blood urea nitrogen
(BUN)
14 mg/dl 10-26
mg/dl
Creatinine
1.0 mg/dl 0.6-1.3
mg/dl
Varies depending on muscle mass, age, and
sex
Glucose
90 mg/dl 70-115
mg/dl
Osmolarity
282 mOsm/l 275-300
mOsm/l
mOsm/l = milliosmoles per liter
Osmolality is expresses as mOsm/kg of
water
Protein, total
7.0 g/dl 6.0-8.0
g/dl
Uric acid
M: 3.0-
7.4
mg/dl
F: 2.1-6.3
mg/dl
Blood Gases
O2 saturation, arterial
98% 95%-99% Percentage of hemoglobin molecules
saturated with oxygen
PO2
, arterial
90 mm Hg 80-100
mm Hg
PO2 = partial pressure of oxygen in
millimeters of mercury
PO2
, venous
40 mm Hg 25-40
mm Hg
PCO2
, arterial
40 mm Hg 35-45
mm Hg
PCO2 = partial pressure of carbon dioxide in
millimeters of mercury
PCO2
, venous
45 mm Hg 41-51
mm Hg
Hematology
Substance
Average
(“Normal”
Value)
Range Comment/Unit of Measure
Hematocrit (Hct)
M: 42% M:
39%-49%
F: 38% F:
35%-45%
Hemoglobin (Hgb)
M: 15 g/dl M: 13.5-
17.5 g/dl
F: 14g/dl F: 12-16
g/dl
Red blood cells (RBCs)
M: 5.5 × 10
8
/µl 4.3-5.7 ×
10
8
/µl
Number of cells per microliter of blood
F: 4.7 × 10
8
/µl 4.3-5.7 ×
10
8
/µl
Mean corpuscular
(RBC) volume (MCV)
90 fl 80-100 fl fl = femtoliters
Prothrombin time (PT)
10-14
seconds
Time required for the plasma to clot during a
special test
Platelets
150-450
× 10
3
/µl
White blood cells, total
4.5-11.0
× 10
3
/µl
Neutrophils
57%-67% Percentage of total white blood cells
Lymphocytes
23%-33% Percentage of total white blood cells
Monocytes
3%-7% Percentage of total white blood cells
Eosinophils
1%-3% Percentage of total white blood cells
Basophils
0%-1% Percentage of total white blood cells
Lipids
Total cholesterol
<200
mg/dl
Substance
Average
(“Normal”
Value)
Range Comment/Unit of Measure
Low-density
lipoprotein (LDL)
<130
mg/dl
High-density
lipoprotein (HDL)
M: >29
mg/dl
F: >35
mg/dl
Triglycerides
M: 40-
160
mg/dl
F: 35-135
mg/dl
This table is not an exhaustive list of common laboratory values. Most of these values are
approximate reference values used by the University of Mississippi Medical Center Clinical
Laboratories; normal ranges may vary among different clinical laboratories. Average
“normal” values and units of measure may also differ slightly from those cited in the Guyton
and Hall Textbook of Medical Physiology, 14th edition. For example, electrolytes are often
reported in milliequivalents per liter (mEq/l), a measure of electrical charge of an electrolyte,
or in millimoles per liter (mmol/l).
guyton and hall physiology review
guyton and hall physiology
guyton and hall physiology review mcq
guyton and hall physiology short notes
guyton and hall physiology review latest edition
guyton and hall physiology review questions pdf
guyton and hall physiology mcqs
guyton and hall physiology review 4th edition
guyton and hall physiology lectures
guyton and hall physiology ppt
guyton and hall physiology anki
guyton and hall physiology anki deck
guyton and hall physiology audiobook
guyton and hall textbook of medical physiology apa citation
guyton and hall medical physiology 13th edition amazon
guyton and hall textbook of medical physiology south asian edition pdf download
guyton and hall textbook of medical physiology 13th edition amazon
guyton and hall textbook of medical physiology 14th edition amazon
guyton and hall physiology south asia edition pdf download
guyton and hall physiology south asia edition pdf
guyton and hall physiology book price
guyton and hall physiology book price in pakistan
guyton and hall physiology book
guyton and hall medical physiology book
guyton hall physiology question book
guyton and hall textbook of medical physiology e-book
physiology by guyton and hall
medical physiology by guyton and hall
guyton and hall textbook of medical physiology book buy
guyton and hall medical physiology review book
guyton and hall physiology chapters
guyton and hall physiology contents
guyton and hall physiology chapter 1
guyton and hall physiology citation
guyton and hall physiology table of contents
guyton and hall textbook of medical physiology citation
guyton and hall textbook of medical physiology chapters
guyton and hall textbook of medical physiology cite
guyton and hall textbook of medical physiology citation apa
guyton and hall textbook of medical physiology content
guyton and hall textbook of medical physiology deutsch
guyton and hall textbook of medical physiology 14th ed
guyton and hall textbook of medical physiology 12th ed citation
guyton and hall medical physiology ebook
guyton and hall human physiology
guyton and hall physiology kit
guyton and hall physiology lecture ppt
guyton and hall physiology latest
guyton and hall textbook of medical physiology latest international edition
guyton and hall physiology medical
guyton and hall medical physiology 14th
guyton and hall medical physiology notes
guyton and hall medical physiology review
guyton and hall textbook of medical physiology 14th edition citation
guyton and hall textbook of medical physiology 12th ed
guyton and hall physiology notes
guyton and hall textbook of medical physiology online
guyton and hall of medical physiology
guyton and hall textbook of medical physiology 14th
guyton and hall textbook of medical physiology 12th edition citation
guyton and hall physiology price
guyton and hall textbook of medical physiology price
guyton and hall textbook of medical physiology ppt
guyton and hall physiology questions
guyton and hall physiology quizlet
guyton and hall physiology review questions
guyton and hall textbook of medical physiology quizlet
guyton and hall textbook of medical physiology quiz
guyton and hall physiology review questions 4th edition pdf
guyton and hall physiology review questions 4th edition
guyton and hall textbook of medical physiology quora
guyton and hall physiology review 3rd edition
guyton and hall physiology review 2nd edition
guyton and hall physiology south asia edition
guyton and hall physiology summary
guyton and hall textbook of medical physiology summary
guyton and hall physiology third south asia edition pdf download
guyton and hall textbook of medical physiology south asian edition
guyton and hall textbook of medical physiology 2nd south asia edition pdf
guyton and hall physiology textbook
guyton and hall medical physiology textbook
guyton and hall physiology videos
guyton and hall textbook of medical physiology videos
guyton and hall textbook of medical physiology tiếng việt
guyton and hall physiology x ray
guyton and hall physiology xr
guyton and hall physiology youtube
guyton and hall physiology zotero
guyton and hall physiology zone
guyton and hall physiology zuid korea
can guyton and hall physiology exam
can guyton and hall physiology edition
can guyton and hall physiology essay
can guyton and hall physiology enzymes
can guyton and hall physiology journal
can guyton and hall physiology jobs
can guyton and hall physiology journal impact factor
can guyton and hall physiology kit
can guyton and hall physiology keys
can guyton and hall physiology of arts
can guyton and hall physiology of medicine
guyton and hall renal physiology
guyton and hall respiratory physiology
can guyton and hall physiology wikipedia
can guyton and hall physiology wiki
can guyton and hall physiology workshop
can guyton and hall physiology x ray
can guyton and hall physiology xr
can guyton and hall physiology youtube
can guyton and hall physiology yacht
can guyton and hall physiology zone
can guyton and hall physiology zurich
can guyton and hall physiology zotero
how guyton and hall physiology essay
how guyton and hall physiology exam
how guyton and hall physiology edition
how guyton and hall physiology endnote
how guyton and hall physiology journal
how guyton and hall physiology journal impact factor
how guyton and hall physiology jobs
how guyton and hall physiology kit
how guyton and hall physiology key
how guyton and hall physiology keys
how guyton and hall physiology of arts
how guyton and hall physiology of medicine
how guyton and hall physiology wikipedia
how guyton and hall physiology wiki
how guyton and hall physiology workshop
how guyton and hall physiology x ray
how guyton and hall physiology youtube
how guyton and hall physiology yacht
how guyton and hall physiology zone
how guyton and hall physiology zurich
how guyton and hall physiology zotero
how guyton and hall physiology zones
introduction to physiology
human physiology
الفيزيولوجيا الطبية
محاضرات physiology
which guyton and hall physiology edition
which guyton and hall physiology exam
which guyton and hall physiology essay
which guyton and hall physiology journal
which guyton and hall physiology jobs
which guyton and hall physiology journal impact factor
which guyton and hall physiology kit
which guyton and hall physiology key
which guyton and hall physiology keys
which guyton and hall physiology of medicine
which guyton and hall physiology of arts
F, Female; M, Male.